Pediatrics Aquifer Questions

Lakukan tugas rumah & ujian kamu dengan baik sekarang menggunakan Quizwiz!

Case 13: A 4-year-old boy who recently emigrated from eastern Europe presents with his mother to your general pediatrics clinic. His mother reports that he has a chronic nonproductive cough during the day and night, mild wheezing for one month and failure to gain weight (his weight has dropped from the 50th to the 10th percentile for his age). His mother denies any high fevers, rhinorrhea, or night sweats. Which of the following are the next best diagnostic tests? A. Chest x-ray and tuberculin skin test B. CT of nasal sinuses C. Spirometry, before and after bronchodilator therapy D. Chest x-ray and methacholine challenge E. None needed, patient likely has habitual cough

.The correct answer is A. A. CXR and tuberculin skin test (TST) is the best choice. Signs and symptoms of primary pulmonary tuberculosis are few to none. Toddlers may present with nonproductive cough, mild dyspnea, wheezing, and/or failure to thrive (defined as weight < 5th percentile or drop in two percentile curves for weight). In children, TB can present without systemic complaints (fever, night sweats, and anorexia), severe cough, and sputum production. Regarding diagnostic tests, the TST is a practical tool for diagnosing TB infections. All children with chronic cough (more than three weeks) should be evaluated with a chest x-ray, as other pathology-such as lung abscess or malignancy-can also be detected on CXR. B. Sinusitis is often preceded by a URI, with nasal congestion as a prominent feature, leading to nocturnal cough due to post-nasal drip. These symptoms are not seen in our patient. Furthermore, a diagnosis of sinusitis is made clinically, with CT scan obtained only in complicated cases or cases resistant to treatment. Complications include cavernous sinus thrombosis, meningitis, and epidural abscess. C. Spirometry (pulmonary function testing) before and after bronchodilator therapy is the most specific means of determining whether or not a child has reactive airways. Asthma is a very common diagnosis in pediatrics, and may present with cough that is worse at night and exacerbated by exercise and cold air. Patients with cough-variant asthma present with only cough, typically nonproductive. However, given this patient's failure to thrive, a more serious diagnosis such as TB must be considered. Also, a chest x-ray is needed in all children with chronic cough (more than three weeks). D. Although a chest x-ray is appropriate in all children with chronic cough, a methacholine challenge (for asthma) would be inappropriate in this scenario. Although asthma is a common diagnosis, given the patient's failure to thrive, a more serious diagnosis must be considered. Further, a methacholine challenge is reserved for cases in which asthma is suspected and spirometry is normal or near normal, and should be performed by trained individuals. E. Habitual cough is caused by habitual perpetuation of a cough that begins with a viral URI. Continued coughing further irritates the airway, leading to stronger stimulation to cough. The cough is typically very loud, short, dry, brassy, and spasmodic. This cough is unchanged by exercise or cold air, and classically resolves during sleep. Although the patient in this case has a dry cough, his failure to thrive points to a more serious diagnosis (e.g., TB). All children with chronic cough (persisting longer than three weeks) need a CXR.

Case 13: An 11-year old boy presents to clinic with wheezing. Mom states that in the past he has used inhaled albuterol and it has helped with wheezing and shortness of breath. On further history you find out that the patient experiences shortness of breath three times a week and is awakened at night by these symptoms once a week. What is the most appropriate outpatient therapy? A. Only rescue inhaler PRN B. Low dose inhaled corticosteroids C. Medium dose inhaled corticosteroids and course of oral corticosteroids D. Medium dose inhaled corticosteroids, LABA, and course of oral corticosteroids E. Course of oral corticosteroids

.The correct answer is B. A. Rescue inhaler (a short-acting beta agonist) i PRN is incorrect because this treatment is indicated in patients with intermittent asthma and have symptoms fewer than two days a week or two nights a month B. Low dose inhaled corticosteroid is correct because this patient has mild persistent asthma. His symptoms occur 3-6 days/week and 3-4 nights/month. C. Medium dose inhaled corticosteroids with a course of oral corticosteroids is incorrect, because it would be indicated in a patient with moderate persistent asthma when symptoms occur daily and more than one night per week. D. Medium dose inhaled corticosteroids, LABA, and oral corticosteroids is incorrect because this patient does not have severe persistent asthma. E. A course of oral corticosteroids alone is incorrect. Asthma needs to be managed long term to prevent exacerbations. An inhaled corticosteroid is indicated.

Case 18: You have accepted a part-time tutoring job for first-year medical students. One of your students asks if you would please clarify the details of normal fetal circulation. Which of the following best describes the path oxygenated blood takes to reach the fetal brain? A. RA > foramen ovale > LA > LV > systemic circulation B. RA > RV > VSD > LV > systemic circulation C. RA > RV > pulmonary circulation > LA > LV > systemic circulation D. RA > RV > ductus arteriosus > LV > systemic circulatio E. RA > RV > ductus arteriosus > systemic circulation

A. Correct. In fetal circulation, the foramen ovale connects the RA to the LA, allowing a portion of the blood to bypass the RV and the lungs. Approximately a third of the blood that enters the RA passes through this route (preferentially the most oxygenated via the unbilical veins which joined the IVC to reach the RA, which is then delivered to the brain and heart), leaving the majority of the blood to travel into the RV. Closure of the foramen ovale is a normal transition from fetal to extrauterine circulation.

Case 27: An 11-year-old male comes to the clinic with a chief complaint of abdominal pain for three months. The pain is not associated with eating. Sometimes he feels full and nauseated, along with the pain, but then it resolves on its own. He denies diarrhea, vomiting, and bloody stools. His mother is primarily concerned because his abdominal pains cause him to miss school quite often now. He is otherwise healthy. When you evaluate his growth curves, he is progressing at the 60th percentile for height and weight and you do not notice a change since birth. He is an honors student. Vital signs are normal. Physical exam, including genital and rectal is normal. Stool is negative for occult blood. What is the most likely diagnosis in this child? A. Functional abdominal pain B. Inflammatory bowel disease C. Bacterial gastroenteritis D. Peptic ulcer disease E. Meckel's diverticulum

A. Functional abdominal pain would be the most likely diagnosis in this setting at this time. History in this setting is not suggestive of any other diagnosis directly causing his abdominal pain, except a change in his social setting. For better understanding of the nature of this child's pain, it would be best to talk to him alone, without his mother present, to determine if he is having trouble adjusting to school and to assess whether he has a stable home environment. His pain is chronic, with no other symptoms (diarrhea, bloody stools, growth failure), making a functional issue most likely.

Case 19: You see a 6-year-old male in the ED who presents with a history of a 10-second episode of jerking movements of his extremities with unresponsiveness, observed by both of his parents. His parents claim he has had abdominal pain and small quantities of bloody diarrhea for two days. The child has no significant past medical history, has taken no medications recently, has no pets, and has not traveled outside of California in the past year. He attends kindergarten. Which organism is the most likely cause of the child's symptoms? The best option is indicated below. Your selections are indicated by the shaded boxes. A. Shigella sonnei B. Rotavirus C. Clostridium difficile D. Enterotoxigenic E. coli (ETEC) E. Vibrio cholerae

A. Shigella sonnei causes bloody diarrhea and WBCs in the stool on Wright stain. Rarely, children infected with Shigella can suffer from seizures due to neurotoxin release.

Case 14: An 18-month-old girl is brought to her pediatrician by her mother who notes that she has been has been fussy for the past three days and has been pulling on her ears. The child is up to date with her vaccines. Her temperature is 102.2 F. Otoscopic exam of her left ear shows a yellow, opaque, and bulging tympanic membrane. Which of the following organisms is the most likely cause of the child's condition? A. Streptococcus pyogenes B. Candida albicans C. Haemophilus influenzae D. Rhinovirus E. Moraxella catarrhalis

A. The child is suffering from acute otitis media (AOM). S. pyogenes is a rare cause of this condition (< 5% of cases). One should not confuse this species of strep with S. pneumoniae, which is a common cause of AOM (25-50% of cases). B. Candida albicans is not a frequent cause of AOM. An infection involving this organism in the middle ear should immediately raise suspicion for an immunocompromised state. C. H. influenzae is a frequent cause of AOM (15-52% of cases). Although the child has been vaccinated against H. influenzae type B, this does not cover the unencapsulated strains of H. influenzae that cause AOM. D. Rhinovirus is a potential cause of AOM, but bacterial AOM occurs with a much higher frequency. E. M. catarrhalis is another common cause of AOM. However, this organism is responsible for only between 3% and 20% of cases, making it a less frequent cause than non-typeable H. influenzae.

Case 20: A 7-year-old boy presents with a five-year history of intermittent vomiting, vertigo, and throbbing unilateral headaches that seem to be induced by emotional stress and when his teacher wears perfume. The headaches occur 1-2 times a month and interfere with his school attendance and lifestyle. He reports that the pain is not worsened by long naps or coughing. His mother reports that she has a history of headaches that started as a child and wonders if her son inherited this from her. His neurological exam shows no focal deficits. What is the next step in diagnosis or treatment? A. Trial of prophylactic medication for migraine headaches B. MRI C. NSAIDs D. Referral to an ENT surgeon

A. This child is presenting with signs of both typical and atypical migraines and could be started on a trial of prophylactic medication. Tricyclic antidepressants (TCAs) are often used in children for migraine prophylaxis, which is the most likely diagnosis in a child with this constellation of symptoms

Case 28: A 20-month-old girl is brought to the primary care clinic for follow-up of cerebral palsy. She was born via home birth at 38 weeks gestation to a primigravda mother. She developed severe jaundice requiring admission at 5 days of life and required phototherapy and exchange transfusion. She has subsequently developed slow and uncontrolled movements throughout her body. Her brain MRI shows atrophy of the basal ganglia. Which type of cerebral palsy is most consistent with this patient's presentation? A. Dyskinetic cerebral palsy B. Spastic diplegia C. Spastic quadriplegia D. Spastic hemiplegia E. Ataxic cerebral palsy

A. This choice is correct because dyskinetic CP is associated with kernicterus, due to hyperbilirubinemia, as well as findings of basal ganglia pathology on imaging. Patients typically have motor abnormalities throughout the body. Dyskinetic CP is also associated with perinatal asphyxia and can involve the thalamus and cerebellum on imaging

Case 28: A 20-month-old girl is brought to the primary care clinic for follow-up of cerebral palsy. She was born via home birth at 38 weeks gestation to a primigravda mother. She developed severe jaundice requiring admission at 5 days of life and required phototherapy and exchange transfusion. She has subsequently developed slow and uncontrolled movements throughout her body. Her brain MRI shows atrophy of the basal ganglia. Which type of cerebral palsy is most consistent with this patient's presentation? A. Dyskinetic cerebral palsy B. Spastic diplegia C. Spastic quadriplegia D. Spastic hemiplegia E. Ataxic cerebral palsy

A. This choice is correct because dyskinetic CP is associated with kernicterus, due to hyperbilirubinemia, as well as findings of basal ganglia pathology on imaging. Patients typically have motor abnormalities throughout the body. Dyskinetic CP is also associated with perinatal asphyxia and can involve the thalamus and cerebellum on imaging.

An 18-month-old boy comes to the clinic for a well-child check. His mother says he is a happy boy and endorses no complaints. She notes that he was born a few weeks early via emergency C-section and spent two months in the NICU. She says he is feeding well and gaining weight. When you ask about developmental milestones, you find out that that he pulls himself up to stand, but is not walking yet. He has several words and is interactive. There is no family history of disease during infancy. He is up to date on his immunizations. He is one of five children and his parents are currently experiencing difficult financial times. Exam reveals increased tone and hyperreflexia in his lower extremities. What is the most likely diagnosis of his developmental delay (if any)?

A. This choice is correct. Children with cerebral palsy often present with defects in motor development and are often found to have abnormal neuromuscular exams, including increased tone and reflexes. Other areas of development are less consistently impacted. The cerebral palsy most likely was the result of an hypoxic injury during the perinatal period.

Case 28:An 18-month-old boy comes to the clinic for a well-child check. His mother says he is a happy boy and endorses no complaints. She notes that he was born a few weeks early via emergency C-section and spent two months in the NICU. She says he is feeding well and gaining weight. When you ask about developmental milestones, you find out that that he pulls himself up to stand, but is not walking yet. He has several words and is interactive. There is no family history of disease during infancy. He is up to date on his immunizations. He is one of five children and his parents are currently experiencing difficult financial times. Exam reveals increased tone and hyperreflexia in his lower extremities. What is the most likely diagnosis of his developmental delay (if any)? A. Cerebral palsy B. Normal variant C. Metabolic disorder D. Genetic abnormality E. Psychosocial stress reaction

A. This choice is correct. Children with cerebral palsy often present with defects in motor development and are often found to have abnormal neuromuscular exams, including increased tone and reflexes. Other areas of development are less consistently impacted. The cerebral palsy most likely was the result of an hypoxic injury during the perinatal period.

A 3-month-old male presents to the ED with a fever that started the previous day. Mother reports that he was fussy and had decreased oral intake. He had had five fewer diaper changes than usual. He had no vomiting, diarrhea, or respiratory difficulty. On physical exam his temperature is 101.6 F, pulse 110 bpm, RR 24 bpm, and BP 95/67 mmHg. The baby seems irritable and is not consolable by the parent. HEENT exam was significant for dry mucous membranes. Other than his irritability, the rest of the physical exam was unremarkable. CBC showed WBC 3.5, but was otherwise normal. BMP was within normal limits. Urinalysis showed positive leukocyte esterase, positive nitrite, and WBCs > 10/hpf. An LP was performed, and urine and CSF culture results are pending. The patient is placed on IV fluids and is started on cefotaxime. What is the next best step in evaluation? The best option is indicated below. Your selections are indicated by the shaded boxes. A. Renal bladder ultrasound B. Kidney-ureter-bladder (KUB) x-ray C. Intravenous pyelogram D. VCUG E. Oral ampicillin

A. This infant has a fever without other respiratory symptoms. Meningitis and UTI must be considered in patients with fever. The only way to rule out meningitis is by lumbar puncture. This patient has a low WBC, suspicious for sepsis, and a UA that is highly suggestive of UTI. Empiric therapy should be started to cover common organisms including E.coli, P. mirabilis, and Klebsiella. Cefotaxime is reasonable empiric therapy. Renal ultrasound is recommended for all infants with pyelonephritis to assess for renal structural abnormalities or signs of obstructive uropathy (hydronephrosis).

Case 19: You are working overnight call in the ED when Charlie, a 3-year-old male infant, arrives after his parents witnessed an episode of convulsions at home. His parents report that Charlie was in his usual state of good health until three days ago when he developed fever, cough, and rhinorrhea. This evening they found him in bed with his eyes rolled upward, jerking all four of his extremities uncontrollably. He was unarousable from this state, which self-resolved after about two minutes. This has never happened before. Currently, Charlie is sleepy but arousable and complains of nausea. His vitals include T 103.2 F, P 112 bpm, BP 100/60 mmHg, RR 22 bpm, O2 sat 99% on room air. Aside from rhinorrhea and erythematous mucous membranes, the remainder of his physical exam is unremarkable. What is the next best step in management? A. Workup for source of fever B. EEG C. MRI brain D. Abdominal ultrasound E. Administration of valproic acid

A. This is the correct answer because Charlie has likely experienced a febrile seizure in the setting of an infection. These are relatively common occurrences in Charlie's age range (6 months to 5 years), and the first priority would be to identify the source of fever and treat it.

Case 7: A one-hour-old newborn male is evaluated in the nursery. He was born to a G2P1 mother at 36 weeks gestation via spontaneous vaginal delivery. The mother did not receive prenatal care because she did not have insurance. She thinks that her water broke about two days ago, but she did not have any contractions after that, so she decided not to come to the hospital. She did not start having contractions until 19 hours before she delivered. No meconium was noted at delivery. Adam did not cry vigorously and was tachypneic and cyanotic. His temperature is 100.5 F. His chest x-ray is normal. Which of the following is the most likely cause of this patient' symptoms? A. Transient tachypnea of the newborn (TTN) B. Sepsis secondary to prolonged rupture of membranes C. Meconium aspiration syndrome D. Hypothermia E. Pneumothorax

A. Transient tachypnea of the newborn (TTN) is a benign, self-limited condition caused by delayed clearance of lung fluid after birth. Patients with TTN usually have a classic chest x-ray that shows coarse fluffy densities that represent fluid-filled alveoli and/or fluid in the pleural space and a small amount of fluid in the fissures on the lateral view. Given Adam's normal chest x-ray and fever, it is unlikely that he has TTN. B. Prolonged rupture of membranes (PROM) is when the chorioamniotic membrane ruptures before the onset of labor. The main risks associated with PROM are preterm labor and delivery and neonatal sepsis. Adam's mom said that her "water broke" two days ago, which indicates that she had PROM. His mother also did not receive prenatal care; therefore, she did not receive any of the prenatal screening tests that she should have, which increases the likelihood that she has an infection that could have potentially been transferred to the infant after the rupture of her membranes. The history of PROM along with his fever and respiratory distress make this answer choice the best choice. C. Meconium aspiration syndrome occurs when the baby passes meconium in utero and aspirates the meconium either in utero or at the time of delivery. Since it was noted that the amniotic fluid did not contain meconium, it is less likely that he is suffering from meconium aspiration syndrome, even though he has symptoms of respiratory distress that can often be seen in meconium aspiration syndrome. D. Hypothermia can cause tachypnea of the newborn, especially premature newborns. However, hypothermia is less likely in Adam given his fever of 100.5 F. E. Pneumothorax is less likely in this case because of Adam's normal chest x-ray, but is important to consider in a tachypneic newborn. The presence of Adam's fever also makes pneumothorax less likely to be the sole cause of his symptoms.

Case 10: A 6-month-old female presents to the Emergency Department because of decreased appetite and fussiness for four days and fever for one day. She had two episodes of non-bilious, non-bloody emesis today with decreased urine output. She was born at 40 weeks gestation. Pregnancy was uncomplicated and this is her first illness. Her vaccinations are up to date. She is non-toxic appearing but fussy. Temperature is 101 F. Her physical examination is non-focal. Which of the following is the most appropriate to evaluate her risk of urinary track infection at this time? The best option is indicated below. Your selections are indicated by the shaded boxes. A. Urinary catheterization B. Renal bladder ultrasound C. Voiding cystourethrogram (VCUG) D. Midstream clean catch urine collection E. Basic Metabolic Panel

A. Urinary catheterization is correct . It is the best method for obtaining a specimen for culture that has not been contaminated by perineal bacteria, and for this ill child, you must determine the cause of the fever with accuracy.

Case 10: A 6-month-old infant arrives in the ED with a 12-hour history of poor feeding, emesis, and irritability. On exam, she is ill-appearing with T 39.2 C, P 160 bpm, R 40 bpm, BP 80/50 mmHg. CBC shows WBC 11.2, Hgb 13.5, Plt 250. Urinalysis shows > 100 WBC per hpf, positive leukocyte esterase, and positive nitrites. She has no history of prior urinary tract infection. Chest x-ray is negative. Urine and blood cultures are pending. After bringing her fever down, she was still uninterested in drinking, but her exam improved, and you were confident she did not have meningitis, so an LP was not performed. Which of the following is the best next step in management? A. Oral ampicillin B. Oral ampicillin + gentamicin C. Intravenous ciprofloxacin D. Intravenous ceftriaxone E. Intravenous piperacillin + tazobactam

Answer D A. This patient is too sick for oral treatment, so oral ampicillin would be insufficient. Also, there is rising resistance of E. Coli to ampicillin, so secondary coverage with gentamicin or some other aminoglycoside would be needed unless cultures proved the organism was sensitive to ampicillin alone. B. Although parenteral and oral treatment produce similar outcomes in high quality RCTs, this patient is ill and refuses to drink and so requires parenteral antibiotics. IV ampicillin and gentamicin could be an appropriate choice for empiric coverage. C. Ciprofloxacin could be used for complicated UTIs, but it has the potential for adverse reactions in young children so is reserved for patients > 1 year with complications such as resistant organisms or urinary tract anomalies D. This patient's presentation is suggestive of a UTI. Given the ill appearance, vital signs, and white count, Upper tract disease (pyelonephritis) should be strongly considered. A parenteral (IV/IM) third-generation cephalosporin is the best choice of those listed for pyelonephritis, given its excellent gram negative coverage (except for Pseudomonas). E. IV piperacillin + tazobactam has excellent gram negative coverage with added Pseudomonas coverage, but it is highly expensive and Pseudomonas is unlikely to be the cause of a UTI in a child who is not regularly catheterized.

Case 8: A concerned mother brings her 7-day-old infant to your office after noticing yellowing of his skin for 2 days. She has also noticed he has not been gaining weight since she brought him home from the hospital 5 days ago. This is her first son and she has been trying to do everything perfectly, including breastfeeding him, since she was told that breast milk provides adequate nutrients and other healthy benefits, like antibodies and growth factors. However, upon further questioning, she is feeding him only 6 times a day for 10 minutes each time. She admits her breasts often feel full and are not relieved by nursing. He was born full term by spontaneous vaginal delivery but had a hard time sucking with breastfeeding. Upon exam, he looks dehydrated and appears to have jaundice of the face and chest. He has also lost > 10% of his birth weight. What could be the cause of his jaundice? A. Breast-milk jaundice B. Physiologic jaundice C. Sepsis D. Breastfeeding jaundice E. Crigler-Najjar syndrome

Answer is D. A. Breast-milk jaundice is incorrect, because although it is a cause of jaundice associated with breastfeeding, it is NOT a result of low milk volume. In the above vignette, the infant does not appear to be breastfeeding well, which is multifactorial-poor suck and low number of feeds for the infant. Breast-milk jaundice would appear only if the infant were feeding well, although it is not completely understood what causes this form of jaundice. B. Physiologic jaundice is incorrect because this jaundice usually peaks at 3-4 days of life to full-term, healthy infants. This type of jaundice is not associated with breastfeeding but could be from numerous factors such as increased bilirubin production from short-lived RBCs or lack of intestinal flora to metabolize bile. No treatment is required for this type and it typically resolves on its own. C. Sepsis is incorrect because the infant does not clinically appear sick or toxic, and jaundice would be just one sign of a serious infection. Other symptoms may include respiratory distress, lethargy, poor feeding, vomiting, apnea, and temperature instability. Sepsis causes an elevated total and direct bilirubin and is highly unlikely when jaundice is the only symptom. Breastfeeding can have some protection against infection. D. Breastfeeding jaundice is the correct answer because it usually appears early in the first week of life and is caused by various factors, including poor breast milk intake. A decreased milk supply leads to limited enteral intake and can lead to increased enterohepatic circulation. Increased enterohepatic circulation describes the process where unconjugated bilirubin is reabsorbed in to the bloodstream where it binds to albumin and is recirculated. E. Crigler-Najjar syndrome is an incorrect choice because it appears within the first days of life and persists thereafter and is a relatively rare disease. This type of jaundice is caused by decreased bilirubin clearance from deficient or completely absent UDPGT (the enzyme that conjugates bilirubin with glucuronide to make it water-soluble and able to undergo excretion into bile).

Case 14: An 8-year-old girl comes to the clinic with two weeks of nasal congestion. Three days ago, she developed a fever of 38.7°C, purulent nasal secretions, malodorous breath, and a nocturnal cough. Examination of the nose reveals bilateral purulent nasal drainage and mild facial tenderness. Which of the following is the most likely diagnosis? A. Allergies B. Maxillary sinusitis C. Asthma D. Frontal sinusitis E. Middle ear infection

Answer: B A. Allergies are a very common diagnosis in pediatrics, and nocturnal cough is a frequently associated symptom. However, allergies usually cause clear, thin nasal secretions and are usually not associated with fever and malodorous breath. Other physical exam findings associated with allergies may include allergic shiners, cobblestoning of the posterior pharynx, and edematous turbinates. B. The maxillary and ethmoid sinuses are large enough to harbor infection in infancy. The sphenoid sinuses do not become large enough until the third to fifth year of life, and the frontal sinuses are rarely large enough until the sixth to tenth year of life. Sinusitis is characterized by the findings in the question stem, and is often preceded by a URI. Pus draining from the middle meatus is suggestive of either maxillary, frontal, or anterior ethmoid sinusitis. C. Asthma, another common diagnosis in pediatrics, is often triggered by a URI, and is frequently associated with a nocturnal cough. Fever, purulent nasal secretions, and malodorous breath are not caused by asthma. End-expiratory wheezing, atopic diathesis, and a positive family history would support a diagnosis of asthma. D. Distinguishing between the different locations of sinusitis can be clinically challenging, especially in a 9-year-old patient who likely has mature sinuses. However, frontal sinusitis is characterized by pain over the frontal bone and perhaps facial swelling in an older child or adolescent. It is less likely to cause the symptoms described here. E. Acute otitis media is a common diagnosis in pediatrics. It can follow a URI or occur simultaneously with sinusitis. However, this patient does not complain of ear pain.

Case 27: A 2-year-old female is brought to the ED by her mother because of increasingly frequent abdominal pain. She has been experiencing this pain on and off for the past year, along with increasing abdominal distention, vomiting, and diarrhea. You chart her height and weight, and find that she is below the 5th percentile for both. IgA tissue transglutaminase (TTG) antibody returns positive. What is the best treatment for this child? A. Antibiotic treatment B. Gluten-free diet C. Corticosteroids D. Pain management E. Metronidazole

B. A gluten-free diet is the best way to manage celiac disease. Celiac disease can present with chronic abdominal pain, vomiting, abdominal distention, and diarrhea. Growth failure can result from malabsorption or anorexia. Anemia may also result from occult GI bleeding, although frank blood in the stool is rare. The IgA tissue transglutaminase antibody titer is a very sensitive and specific test for this disease.

Case 19: A previously healthy and developmentally normal 16-month-old male comes to the urgent care clinic with his father with a chief complaint of his first reported seizure. The child was reported to have dropped to the floor with loss of consciousness and had sporadic twitchy movements of his legs and arms that lasted for five minutes. The child has had URI symptoms for the past two days, with a fever to 103 degrees F without any changes in mental status. Neither parent has a seizure disorder, but the child's mother reports having a single seizure as a young girl once after developing a high fever after a cold. What is the most likely diagnosis? A. Epilepsy B. Simple febrile seizure C. Complex febrile seizure D. Absence seizure E. Cyanotic breath-holding spell

B. Choice B is correct. Febrile seizures are one of the most common causes of seizures in children. Simple febrile seizures are more common than complex febrile seizures and are characterized by < 15 minutes duration, occurring only once in a 24-hour period, and are generalized (in this patient's case, generalized tonic-clonic). Febrile seizures are usually self-limited events triggered by an acute febrile illness. A positive family history for febrile seizures in the parents makes it a more likely diagnosis in their children. This particular diagnosis fits this patient the best.

Case 26: A 6-month-old infant is referred to the pulmonology clinic because he was recently diagnosed with cystic fibrosis. He was born in Mexico and immigrated to the United States 4 months ago. Due to poor weight gain evaluation by his primary care provider revealed an elevated sweat chloride level. Which of the statements about cystic fibrosis is most correct? The best option is indicated below. Your selections are indicated by the shaded boxes. A. CF is an autosomal dominant disorder B. CF is caused by a mutation in CFTR, resulting in defective salt balance C. CF is a disease that exclusively involves the respiratory system D. Gene therapy is now the primary source of CF therapy E. It is important to provide calories at a lower level than recommended dietary allowance for a given age in order to prevent GI upset

B. Correct. The mutation in CFTR gene results in defective salt balance. CF is caused by mutations in a single large gene on chromosome 7 that encodes the cystic fibrosis transmembrane conductance regulator (CFTR) protein. Clinical disease requires disease-causing mutations in both copies of the CFTR gene.

Case 12: Susie is a 3-year-old girl brought into the clinic by her mother because she has a gradually worsening cough and she has been having trouble breathing. Her mother says Susie sounds like she is barking when she coughs. Susie is up to date with her vaccinations. Susie's mom always watches her when she's playing. On physical exam, you note that Susie has inspiratory stridor. She does not have wheezing, there are no retractions, and she has symmetrical breath sounds. No pseudomembranes are appreciated on physical exam. What is Susie's most likely diagnosis? A. Epiglottis B. Croup (laryngotracheobronchitis) C. Pertussis D. Pneumonia E. Foreign body aspiration

B. Croup or laryngotracheobronchitis is due to a viral infection (Parainfluenza type 1). It is most common in the winter, and often occurs in children age 2 to 5 years. Croup can lead to non-specific URI symptoms with some degree of airway obstruction. A barky or seal-like cough and inspiratory stridor (which should be differentiated from expiratory wheezes) is common in croup.

A 3-day-old female is evaluated in the nursery for poor feeding since birth. The infant takes 1 ounce of formula every 4 hours. Each feed takes about 40 minutes because the patient falls asleep during the feed. The patient has vomited twice today and stooled once yesterday. Physical exam reveals a lethargic infant who is difficult to arouse by exam. The liver is enlarged and muscle tone is decreased. Serum ammonia level is elevated. Which of the following laboratory results would be expected in this patient? A. Polycythemia B. Elevated urine orotic acid C. Elevated 17-OH progesterone D. Elevated thyroid stimulating hormone E. Hyponatremia

B. Elevated urine orotic acid is diagnostic of OTC deficiency, an x-linked condition, the most common urea cycle disorder. This diagnosis is most likely based on the enlarged liver, mental status changes and hyperammonemia.

Case 26: An 8-week-old infant is brought to the clinic for a health maintenance visit. She was a twin born at 40 weeks gestation. Pregnancy and delivery were uncomplicated. At birth her weight, height, and head circumference were at the 50th percentile; she is now at the 5th, 10th, and 25th percentiles, respectively. She and her twin sister are exclusively breastfed every four hours scheduled. She does not have any vomiting and stools about 3 times a day. Physical exam reveals a thin but otherwise healthy infant. What is the most likely cause of this infant's growth pattern? The best option is indicated below. Your selections are indicated by the shaded boxes. A. Congenital heart disease B. Inadequate caloric intake C. Cystic fibrosis D. Milk protein allergy E. Inborn error of metabolism

B. Inadequate caloric intake is correct because it is the most common cause of FTT. Furthermore, we have reason to believe that this particular infant is not receiving adequate calories. Breastfed infants should eat every two to three hours until 3 months of age; this infant eats every four hours and shares the milk supply with her twin. Twins can thrive on exclusive breastfeeding, but it requires that the mother consume extra fluids and calories and ensure that the twins both get enough to eat.

Case 28: An 18-month-old boy is brought to the primary care office for a well child visit. His family recently moved into town, and this is the first visit to the clinic. He was born at 32 weeks gestation and spent 4 weeks in the nursery due to breathing problems. Developmental screening shows normal social language and social development, but delayed gross motor development. He is able to stand, but not walk. Physical exam reveals increased tone, exaggerated deep tendon reflexes, and clonus in both of his lower extremities. Which of the following is the most likely diagnosis in this patient? A. Autistic spectrum disorder B. Cerebral palsy C. Metabolic disorder D. Chromosomal disorder E. Myopathy

B. Spastic diplegia is a form of cerebral palsy, a non-progressive static encephalopathy characterized by delays in motor development. It may be associated with periventricular white matter abnormalities that are thought to be due to ischemia. These changes can be visualized on MRI. In spastic diplegia, the motor abnormalities are often greater in the legs than in the arms.

Case 28: An 18-month-old boy is brought to the primary care office for a well child visit. His family recently moved into town, and this is the first visit to the clinic. He was born at 32 weeks gestation and spent 4 weeks in the nursery due to breathing problems. Developmental screening shows normal social language and social development, but delayed gross motor development. He is able to stand, but not walk. Physical exam reveals increased tone, exaggerated deep tendon reflexes, and clonus in both of his lower extremities. Which of the following is the most likely diagnosis in this patient? The best option is indicated below. Your selections are indicated by the shaded boxes. A. Autistic spectrum disorder B. Cerebral palsy C. Metabolic disorder D. Chromosomal disorder E. Myopathy

B. Spastic diplegia is a form of cerebral palsy, a non-progressive static encephalopathy characterized by delays in motor development. It may be associated with periventricular white matter abnormalities that are thought to be due to ischemia. These changes can be visualized on MRI. In spastic diplegia, the motor abnormalities are often greater in the legs than in the arms.

Case 20: A 12-year-old girl presents to her pediatrician complaining of a headache of gradual onset x 3 hours, non-provoked and described as a "big rubber band around my whole head" and a 5 out of 10 on the pain scale. The pain is not throbbing, and there is no associated photophobia, nausea or vomiting. The patient is afebrile, and there are no neurologic deficits during physical exam. Her mother states her pain is typically relieved with ibuprofen, but her mother is concerned that patient may have migraines because she has a few headaches every month after school. The child is otherwise healthy. What is the most likely cause of this girl's headaches? A. Migraine B. Tension-type headache C. Brain tumor D. Sinusitis E. Idiopathic intracranial hypertension

B. Tension-type headaches are often bilateral and involve the forehead, temporal areas, or back of the head. Tenderness of the posterior muscles of the neck may also be present. They should be responsive to NSAIDs. Stress can give rise to a tension headache, and this is consistent with this patient developing headaches after school.

Case 12: A 10-month-old infant is brought to the Peds ED by her parents, who say she has been coughing persistently for the last three hours. The parents were watching a movie at home when they first noticed their daughter coughing. Patient is a vaccinated, well-nourished infant in moderate distress with retractions, nasal flaring, and grunting. On auscultation, you immediately notice diminished breath sounds in the right lung with normal breath sounds on the left. What other associated physical exam finding do you expect to hear? A. Stridor B. Asymmetric breath sounds and wheezing C. Rhonchi D. Crackles E. Bronchial breath sounds

B. This infant is in respiratory distress from foreign body aspiration, consistent with the history of acute onset of distress and asymmetric breath sounds. Common foreign bodies include peanuts, popcorn, grapes, hard candy and hot dogs. Respiratory distress from foreign body aspiration is usually accompanied by asymmetric breath sounds and wheezes on auscultation.

A 10-month-old infant is brought to the emergency room by his very concerned and frantic grandmother. Earlier that day, she retrieved the child from his mother's new boyfriend, who had been watching him while his mother was at work. The grandmother makes it very clear she does not approve of this new boyfriend, and she is concerned that he is rough with her grandson. She demands that her grandson be worked up for injuries and that a restraining order be placed against the boyfriend. Which of the following finding does NOT indicate that a child is being physically abused? The best option is indicated below. Your selections are indicated by the shaded boxes. A. Retinal hemorrhages on fundoscopy B. A concaved, crescent-shaped mass on head CT C. A spiral fracture of the tibia D. Two posterior rib fractures E. A metaphyseal fracture of the wrist

C. Also called a "toddler's fracture," fracture of the tibia is a commonly occurring fracture in young, ambulatory kids. It is not a sign of abuse. Toddler's fracture is described as a subtle, non-displaced oblique fracture of the distal tibia in kids aged 9 months to 3 years. The child will usually present with acute onset of limp and refusal to bear weight on one leg. It usually occurs when a toddler falls while twisting, or gets a foot caught and falls while trying to free the foot.

Case 27: Kenny is a 12-year-old boy who comes to your clinic with a chief complaint of crampy abdominal pain. His mother tells you that sometimes he wakes up from sleep due to the pain. He also has diarrhea that sometimes has blood in it. When asked about stressors in his life, his mother sighs and tells you that she is recently divorced. On physical exam, he appears small for his age. Abdomen is soft, non-distended, but tender to palpation at the RUQ. On rectal exam, you note anal skin tags and an anal fistula. Skin exam shows red tender nodules on his shins. What is the next best step in management? A. Reassure and refer to psychiatry B. Start omeprazole and antibiotics C. Colonoscopy D. Start mesalamine E. Obtain IgA endomysial antibody and IgA anti-tissue transglutaminase antibody

C. Colonoscopy is the best answer. Kenny likely has IBD (Crohn's disease or ulcerative colitis [UC]). He has crampy abdominal pain and intermittently bloody diarrhea, crampy abdominal pain (that wakes him up at night), perianal disease, and erythema nodosum. He also has microcytic anemia, likely from chronic blood loss. Colonoscopy with biopsies will allow you to diagnose Crohn's disease (or UC) prior to treating it. The diagnosis begins with a colonoscopy to obtain tissue biopsies as well as blood tests (p-ANCA, ASCA).

Case 9: A two-month-old infant is brought by her mother to clinic for a well-baby checkup. Mom says that her daughter is easy to care for because "she rarely cries and sleeps most of the time." On exam, the patient has a yellow tint to the skin, decreased muscle tone, and a large anterior fontanel. What is the most likely diagnosis in this patient? The best option is indicated below. Your selections are indicated by the shaded boxes. A. Sepsis B. Congenital adrenal hyperplasia C. Congenital hypothyroidism D. Shaken baby syndrome E. Neonatal lupus

C. Congenital hypothyroidism may not be clinically evident until 6 weeks of age due to circulating maternal thyroid hormone transmitted from the placenta. Signs and symptoms of congenital hypothyroidism include feeding problems, large fontanelles, hypotonia, large tongue, coarse cry, and frequently an umbilical hernia. Congenital hypothyroidism should be picked up on routine neonatal screening.

Case 28: A 15-month-old is able to stand on his own, walk backward, and throw objects underhand. He is unable to draw/scribble or grasp markers. He can say one syllable words "ma" and "da" but his words are unintelligible. He holds a sippy cup with help. He does come when called, plays with a ball and waves bye-bye by imitating his parents. Which of the following is the most likely diagnosis in this patient? A. Normal development B. Gross motor and fine motor delay C. Fine motor and language delay D. Language delay and gross motor E. Social and language delay

C. Correct. A 15-month-old child should be able to scribble, use a cup, and/or stack 2 blocks. His word count should be three to six words at this stage of development

A 15-month-old is able to stand on his own, walk backward, and throw objects underhand. He is unable to draw/scribble or grasp markers. He can say one syllable words "ma" and "da" but his words are unintelligible. He holds a sippy cup with help. He does come when called, plays with a ball and waves bye-bye by imitating his parents. Which of the following is the most likely diagnosis in this patient?

C. Correct. A 15-month-old child should be able to scribble, use a cup, and/or stack 2 blocks. His word count should be three to six words at this stage of development.

Case 18: A 5-year-old boy is noted to have a grade II systolic murmur and a widely split S2 murmur on cardiac exam. His vital signs are stable and he has been asymptomatic. Which of the following statement is accurate regarding this child's presentation and likely condition? A. No further work-up for a presumed venous hum B. Chest x-ray, ECG, and echocardiogram would be indicated as next steps to work up a presumed ventricular septal defect C. This patient's murmur is caused by flow through the pulmonary outflow tract and should be evaluated D. The patient should be scheduled now for cardiac catheterization

C. Correct. This patient's murmur is likely caused by an atrial septal defect, which causes flow of additional blood through the pulmonary outflow tract and should be evaluated.

Case 25: An 18-year-old mother with her 3-month-old son arrives at urgent care clinic with a chief complaint of "my baby stopped breathing!" She and her baby are rushed into a triage room, where her son is noted to be very lethargic with increased work of breathing. As vital signs are being obtained, the mother reports "my baby stopped breathing in the car coming here, and didn't start again until I reached over and jostled his car seat!" Mom then shared that "my boyfriend said he rolled off the couch last night. Could it be related?" Mom also stated that her son hasn't been as active as usual, and has been vomiting occasionally. Physical exam is notable for a respiratory rate of 70 bpm with intercostal retractions and crackles in the right lower lung fields posteriorly. You admit this patient with the diagnosis of pneumonia for empiric antibiotics and support, pending additional evaluation. CXR subsequently demonstrates a RLL infiltrate and faint, vertical lines on several posterior ribs bilaterally. What is the best next step in management? A. Obtain a PTH level B. Sweat chloride testing C. Skeletal survey (more x-rays) D. Anticipatory guidance about appropriate car seat usage E. Head ultrasound

C. In this case a skeletal survey is essential. Posterior rib fractures are always concerning findings. While treatment of the patient's pneumonia has been initiated, a complete skeletal survey will screen for other worrisome findings, including multiple fractures in different stages of healing, fracture of the femur or tibia in a non-walking child, and skull fractures.

Case 20: A 7-year-old boy with a past medical history of headaches presents with increased frequency and severity of headaches along with new onset vomiting. When the boy was walking into the room, he had a wide stance and nearly tripped twice. Which of the following is the most appropriate next step? A. Daily headache diary B. Computed tomography C. Magnetic resonance imaging D. Lumbar puncture E. Intraventricular pressure monitoring

C. MRI is more expensive and less readily available than CT imaging. It also frequently requires sedation in pediatric patients. However, it provides the best detail of the posterior fossa, which is the most common location of pediatric brain tumors.

Case 12: A 12-year-old boy presents to the ED with complaints of anorexia, weight loss, and persistent cough, with nocturnal coughing fits that have been waking him from sleep for the past three weeks. He denies fever, chills, myalgia, sore throat, or rhinorrhea. The patient presented to his primary care physician one week prior with the same complaint, and was treated with amoxicillin and bronchodilator therapy. His chest x-ray was negative for infiltrates at that visit. The patient's symptoms did not improve with this regimen. The cough became more frequent, sometimes causing emesis. Which of the following is the most likely diagnosis? A. Reactive airway disease B. Infection with Bordetella pertussis in the catarrhal stage C. Infection with Bordetella pertussis in the paroxysmal stage D. Atypical pneumonia due to Mycoplasma pneumoniae E. Laryngotracheobronchitis

C. The paroxysmal stage of pertussis lasts four to six weeks and is characterized by repetitive, forceful coughing episodes, followed by massive inspiratory effort. This massive inspiratory effort is what results in the characteristic "whoop"-sounding cough. This is consistent with the patient's presentation and duration of illness. The forceful coughing fits in pertussis can even lead to conjunctival hemorrhages and pneumothoraces from the increased intrathoracic and intracranial pressures from Valsalva. The antimicrobial agents of choice for treatment of pertussis are azithromycin, clarithromycin, and erythromycin. Antibiotics given in the paroxysmal phase will reduce communicability but will not alter the clinical course.

Case 19: A woman brings her 8-year-old son to the pediatrician after witnessing him stare blankly into the distance at dinner the previous week. He was unresponsive to her calling his name or any other stimuli, and it lasted for about 10 or 20 seconds. His teacher reports he does seem to daydream often in class but is able to keep up with schoolwork and excels in his studies. She doesn't note him being disruptive or impulsive in class. His mother is concerned about these blank stares and unresponsive episodes. Which of the following is the most likely diagnosis? A. Generalized tonic-clonic seizure B. Atonic seizure C. Absence seizure D. Simple partial seizure E. Complex partial seizure

C. The patient is having absence seizures. Absence seizures are characterized by loss of awareness of surroundings ("blank stare" or "in another world") and automatisms (e.g., eye-fluttering or lip-smacking). These patients do not lose consciousness or have loss of tone. Absence seizures should also be differentiated from ADHD, since children with ADHD also can be inattentive or seem to be daydreaming. However, since the patient in the vignette still does well in school and does not have other signs of ADHD, it is most likely an absence seizure. An EEG will confirm the diagnosis.

Case 25: A 2-month-old infant is brought to the ED after his mother found him in his crib not breathing. She says he had no color and was still when she found him, but quickly regained his color. While you are examining him he starts having a tonic-clonic seizure and subsequently is found to have a temperature of 96 F, HR 200 bpm, and RR 18 bpm. On exam he cries intermittently, does not track you with his eyes, has a tense, full fontanelle, and decreased tone throughout. You also notice a healing bruise on his left arm. After assessing circulation, airway, and breathing you obtain IV access. What is the next step in your diagnostic workup? A. Skeletal survey B. Lumbar puncture C. Head CT D. Head MRI E. Social work consult

C. This choice is correct because head CT is highly sensitive for an intracranial bleed, such as a subdural hematoma, can be quickly carried out in the emergency setting, and may require urgent intervention.

A 6-week-old infant is brought to the emergency room by his mother due to sleepiness, constipation, and yellow skin for the past 3 weeks. The mother and the baby recently immigrated to the United States from Malaysia. No medical records from the delivery are available. The infant has been breast-fed since birth. Vital signs are normal. Physical examination reveals jaundice, large anterior and posterior fontanelles, a large tongue, and abdominal distension. Which of the following is the most appropriate next step in management of this patient? A. Phototherapy B. Exchange transfusion C. Thyroid function studies D. Head ultrasound E. Family history

C. This choice is correct because the constellation of baby's problems is best accounted for by untreated congenital hypothyroidism. Unfortunately, severe mental retardation is unavoidable at this point because this condition should have been treated since birth. In the U.S., it would have been detected on the newborn screen.

Case 26: A two-week-old infant presents to the clinic to follow-up on abnormal newborn screening results. The routine newborn screen showed increase immunoreactive trypsinogen which is concerning for cystic fibrosis. Confirmatory testing is sent. What signs and symptoms should be monitored in this infant? A. Bloody stools B. Precordial thrill C. Reflux D. Increased stooling E. Diaphoresis during feeds

D. CF causes exocrine insufficiency and ultimately results in fat malabsorption. This causes frequent loose stools in infancy and progresses to foul-smelling and greasy stools in childhood.

Case 19: During the middle of dinner on your day off, you receive a call from one of your neighbors who remembers that you are a medical student. He is concerned about his 15-year-old daughter who had previously been in her usual state of health and has no significant past medical history. However, over the past 24 hours, his daughter suddenly spiked a fever of 103 F and has "not been herself," acting very lethargic and dazed at times. He also notes that she has been breathing heavily, not been able to eat or drink, and has not urinated over the past 12 hours. He wants your advice about whether she should be taken to the ED. Although you are fairly certain that the best course of action would be to take her to the ED, you contemplate the differential diagnosis of her presentation. Given the limited history, which of the following is highest on your differential? A. Acute cystitis B. Diabetic ketoacidosis C. CNS tumor D. Meningitis E. Hypoglycemia

D. Choice D is correct because meningitis is the most likely etiology in our differential given the fever, altered mental status, decreased PO intake, and decreased urine output. While the incidence of meningitis has decreased in this patient's age range due to increased vaccinations against the most common causative organisms of meningitis, it still remains high on our differential given the presentation of this patient. In the ED, we would likely need to obtain a more thorough history and physical exam as well as blood cultures and lumbar puncture to establish the diagnosis of meningitis.

A young couple presents to the ED with their 2-month-old son complaining of excessive sleepiness and difficulty arousing him after his nap. Per the parents, the PMH and prenatal course are unremarkable, except that the patient has always been very fussy and would often cry despite being held and cradled. He is cared for during the day by his babysitter. Today he had been in his usual state of fussiness when the babysitter arrived, and they returned to find him napping quietly in his cradle but could not arouse him from sleep when it came time for his feeds. He finally opened his eyes after several minutes of gentle nudging but seemed to quickly fall asleep again. On exam, the infant is afebrile with poor tone and is only mildly responsive to painful stimuli. Eye exam shows dilated pupils and an ophthalmology consult reveals retinal hemorrhages. What is the most likely diagnosis? A. Bacterial meningitis B. Infant botulism C. Intoxication D. Closed head injury E. Metabolic disorder

D. Closed head injuries and retinal hemorrhages in infants and children are secondary to violent shaking or throwing, resulting in tearing of the bridging vessels. Retinal hemorrhages, as seen in our patient, are pathognomonic for shaken baby syndrome. Other signs and symptoms include stiffness, constant crying, seizures, difficulty to arouse, decreased appetite and excessive sleeping. Victims often have no other signs of physical abuse (e.g., bruises).

A 6-week-old infant is brought by her parents to the clinic for increased sleepiness for the past 2 weeks. The patient is not easily aroused for feedings and is not as alert as she was previously. The infant was born in Honduras and the family recently immigrated to the United States. Vital signs are normal. Physical exam reveals jaundice, an enlarged fontanelle and an umbilical hernia. Neonatal screening was not performed. Which of the following laboratory values would most likely be abnormal in this patient? The best option is indicated below. Your selections are indicated by the shaded boxes. A. Low sodium, high potassium B. Glucose < 40 mg/dL C. High WBC with bandemia D. High TSH, low T4

D. Congenital hypothyroidism may present with poor feeding, constipation, jaundice (longer and more persistent than physiologic jaundice of newborn), mottled skin, large fontanelles, hypotonia, hypothermia and an umbilical hernia. Later findings include a hoarse cry, macroglossia, and myxedematous facies. Patients usually remain asymptomatic until after 6 weeks of age, as maternal thyroid hormones may still be in younger infants. Patients with primary hypothyroidism will have high TSH and low T4 levels. The most common cause of primary hypothyroidism will be aplasia or hypoplasia of the thyroid gland, and-much less commonly-inborn errors of metabolism. Secondary or tertiary hypothyroidism (HPA dysfunction) will have both low or inappropriately normal TSH and low T4, and are relatively rare causes of hypothyroidism in infants.

Case 18: A 5-week-old infant is brought to the pediatrician for failure to thrive (despite adequate, even prolonged, feedings) and respiratory distress (particularly tachypnea). What other features does this infant most likely have? A. Cyanosis from a right-to-left shunt B. Systolic murmur with a widely split second heart sound C. An early systolic click without a murmur D. A hyperdynamic precordium with a holosystolic murmur

D. Correct. A heart murmur from a VSD is typically not appreciated in the immediate newborn period, as the pulmonary vascular resistance is still quite elevated. During this time, since the pulmonary vascular resistance equals the systemic vascular resistance, there is no shunting of blood through the open VSD. However, after a few days to weeks after birth, the pulmonary vascular resistance decreases, and the murmur appears, reflecting the shunted flow of blood through the open VSD (from left to right). As flow through the VSD increases and becomes audible, the child is at risk for developing symptoms related to excessive pulmonary flow: tachypnea and poor weight gain.

Case 27: An 8-year-old girl is brought to clinic because of intermittent, crampy abdominal pain that has persisted over the last three months. The pain is nonspecific, nonfocal, and not associated with any other systemic symptoms such as fever, chills, weight loss, nausea, vomiting or diarrhea. The pain also seems to occur more frequently during the week and not as often on weekends. The abdominal exam is normal. Jenny is given a diagnosis of functional abdominal pain. Six months later, she returns to the clinic complaining of more frequent, more severe abdominal pain that is waking her up at night. She also reports a week of diarrhea containing mucus and blood without associated fever or vomiting. Review of her growth chart demonstrates a slowing of weight gain and a drop in height velocity. What is the most likely diagnosis? A. Irritable bowel syndrome B. Giardiasis C. Celiac disease D. Crohn's disease E. Henoch-Schonlein purpura (HSP)

D. Crohn's disease is most consistent with this presentation, as it affects GI tract from mouth to anus, leading to abdominal pain, diarrhea (can be bloody), vomiting, or weight loss. Extraintestinal symptoms include skin rashes, arthritis, and fatigue. Fever, fistula, and perianal complications are also common.

Case 12: A 12-month-old previously healthy girl presents with cough and mild subcostal retractions. She is afebrile, and physical exam reveals asymmetric wheezing. Chest x-ray demonstrates unilateral air trapping. What is the most likely diagnosis? A. Croup B. Pneumonia C. Acute bronchiolitis D. Foreign body aspiration E. Asthma

D. Features of foreign body aspiration include unexplained wheezing and asymmetric breath sounds, as well as air trapping in one lung indicating unilateral airway obstruction. The right main bronchus is the more commonly obstructed due to anatomy (it is wider and more vertical than the left). The most commonly aspirated foods are hot dogs, nuts, hard candy, grapes, and popcorn.

A 10-day-old boy is brought to the ED by his mother because of "fever." Mom describes that the baby has been "sleepy" and feeding less vigorously than in the previous two days. She believes his urine output has also decreased. His birth history is notable for prolonged membrane rupture (about 32 hours), and maternal fever at the time of delivery. Prenatal and neonatal ultrasound revealed bilateral hydronephrosis. On exam, the infant is sleepy with a temperature of 38.5 C. A blood sample is sent for CBC, BMP, and culture. Attempts are made to obtain CSF and urine for analysis and culture, but only very small volumes of these fluids are obtained. Volume resuscitation is begun. Chest x-ray is performed with indeterminate results. What is the most appropriate next step? A. Send samples for gram stains and begin parenteral empiric antibiotic treatment B. Send the urine for urinalysis and the CSF for cell count, glucose and protein and begin parenteral antibiotic therapy C. Admit for observation and continue supportive care D. Send samples for culture and begin parenteral antiobiotic treatment E. Attempt to obtain larger samples. Antibiotics should not be started until all needed results are pending.

D. Given the presentation of fever in a neonate who presents with sleepiness and poor feeding, samples should be sent for culture and the baby started on empiric antimicrobial therapy. This infant is likely to have a urinary tract infection, and urosepsis is certainly a possibility, especially given his known urinary tract anoamlies. We have no way of ruling out meningitis from this presentation, so antibiotics should be initiated at meningitic dosing. In an infant younger than one month, fever with any suspicion of sepsis, whatever the source, requires immediate evaluation and initiation of antibiotic treatment. Because infants at this age have immature immune systems, they do not localize infections as well as older children. An infection of the urinary tract may lead to bacteremia, which in turn may lead to CNS infection. Only cultures will give us the information required to determine the appropriate type length of antimicrobial therapy.

Case 8: A 5-day-old infant presents with a chief complaint of jaundice. As you obtain a careful history and physical examination, which of the following would NOT be a risk factor for jaundice in this infant? A. Mediterranean origin B. Prolonged labor with use of forceps during the delivery C. Mother is type O+ and baby is type B D. Phenylketonuria E. Poor breastfeeding during first few days of life

D. Phenylketonuria (PKU) is an autosomal recessive metabolic disorder due to a mutation in phenylalanine hydroxylase, which is required to convert phenylalanine to tyrosine. PKU leads to buildup of phenylalanine in the brain, leading to mental retardation, seizures, and death if not detected and treated early. It is not associated with jaundice.

A 3-year-old girl presents to the ED with sudden onset difficulty walking. She does not have a fever, headache, nausea, or vision changes, but two weeks ago she had a runny nose, a fever, and a rash. Musculoskeletal exam reveal no abnormalities of lower extremities. Neurologic exam reveals bilateral horizontal nystagmus, wide based stance and swaying, and bilateral overreaching on finger to nose test. An LP is performed which reveals a normal CSF. Which of the following is the most likely diagnosis? A. Opsoclonus-myoclonus syndrome B. Hydrocephalus C. Infectious cerebellitis D. Post-infectious cerebellitis E. Migraine headache

D. Post-infectious cerebellitis is correct. This typically presents in a younger child with ataxia, nystagmus, vomiting and sometimes dysarthria. It is believed to be an autoimmune response leading to demyelination of the cerebellum occurring several weeks after a viral infection such as varicella or coxsackie virus.

Case 26: A 6-week-old is brought into clinic by his mother for poor weight gain. He was born at 40 weeks gestation. Pregnancy and delivery were uncomplicated. Family history reveals a cousin with trouble gaining weight and frequent "lung infections." He drinks four ounces of formula every two to three hours. He does not spit-up during feeds. He stools about 10 times a day. His weight today is the same as at his 2-week visit. His weight is < 5th percentile, and height and head circumference are at the 25th percentile. Vital signs are normal. Physical exam reveals decreased adipose tissue, but is otherwise normal. Which of the following is the best next step in management of this patient's growth? A. Increase formula to a higher calorie mixture B. Stool culture and Wright stain C. Swallow study D. Sweat chloride test E. Echocardiogram

D. The patient's greasy and foul-smelling stool is characteristic of steatorrhea, or fat in the stool. This occurs in patients with cystic fibrosis due to their impaired pancreatic exocrine function. Given the stool history in the face of failure to thrive, a sweat chloride test would be indicated at tis time. Of note, the family history of poor weight gain in a cousin with frequent lung infections also suggests the possibility of cystic fibrosis-related bronchiectasis. Cystic fibrosis should still be suspected in the child who has a normal newborn screen. States differ in the type of test offered to screen for cystic fibrosis, and although great strides have been made in newborn detection, methods are not 100% sensitive.

Case 9: A 2-week-old infant is brought by her mother to the clinic because of concerns for jaundice, constipation, sleepiness, and poor feeding. She has not had any vomiting. The patient was born at home and received no medications or lab studies. Physical exam reveals enlarged anterior fontanelle, jaundice, hypotonia, and an umbilical hernia. The remainder of the examination is normal. After confirmation of the diagnosis, which of the following is the most appropriate pharmacotherapy in this patient? The best option is indicated below. Your selections are indicated by the shaded boxes. A. Glucose and electrolyte supplementation B. Glucocorticoid and mineralocorticoid supplementation C. No treatment needed D. Consult with pediatric endocrinologist and start treatment with 10 to 15 mcg/kg/day of crushed levothyroxine in liquid, and follow up every 12 months E. Empiric antibiotics after collection of blood, urine, and CSF cultures

D. This choice is correct because the American Academy of Pediatrics recommends this treatment regimen for infants age 0 to 6 months old. Dosing is based upon age and weight. It would also be important to consult with a pediatric endocrinologist to evaluate the short and long-term treatment plan.

A 5-month-old is brought by her parents to the clinic because of decreasing oral intake over the past 4 days. The patient has been sleeping more than previously and seems to tire out when feeding. The patient breast feeds and eats home-made pureed vegetables. The patient has not had a bowel movement in 3 days. She has no fever or respiratory symptoms. Physical examination reveals a weak cry and decreased strength. What additional physical examination findings would be expected with this patient's presumptive diagnosis? A. Vesicular rash on her scalp B. Large tongue and umbilical hernia C. III/VI systolic murmur D. Absent deep tendon reflexes E. Cataracts and hepatosplenomegaly

D. This infant likely has infant botulism which usually presents in the first year of life with hypotonia, lethargy, constipation, weak cry and can eventually lead to respiratory failure. These infants will have absent DTRs.

Case 10: A 6-month-old infant arrives in the ED with a 12-hour history of poor feeding, emesis, and irritability. On exam, she is ill-appearing with T 39.2 C, P 160 bpm, R 40 bpm, BP 80/50 mmHg. CBC shows WBC 11.2, Hgb 13.5, Plt 250. Urinalysis shows > 100 WBC per hpf, positive leukocyte esterase, and positive nitrites. She has no history of prior urinary tract infection. Chest x-ray is negative. Urine and blood cultures are pending. After bringing her fever down, she was still uninterested in drinking, but her exam improved, and you were confident she did not have meningitis, so an LP was not performed. Which of the following is the best next step in management? A. Oral ampicillin B. Oral ampicillin + gentamicin C. Intravenous ciprofloxacin D. Intravenous ceftriaxone E. Intravenous piperacillin + tazobactam

D. This patient's presentation is suggestive of a UTI. Given the ill appearance, vital signs, and white count, Upper tract disease (pyelonephritis) should be strongly considered. A parenteral (IV/IM) third-generation cephalosporin is the best choice of those listed for pyelonephritis, given its excellent gram negative coverage (except for Pseudomonas).

A 6-month-old female is brought into the pediatrician's office for three days of high fever, fussiness, and decreased appetite. The patient has not had any upper respiratory tract symptoms, vomiting, diarrhea, or rash. On physical exam the patient is fussy, has a RR of 28 bpm and a pulse of 160 bpm. She is febrile to 102.8 F (rectal). The patient is alert and fully moving all extremities. Apart from her vital signs, no other significant exam findings are noted. A CBC demonstrates leukocytosis of 17.0 cells x 103 / µL with elevated bands. What diagnosis is most likely? A. Measles B. Bacterial meningitis C. Acute otitis media D. Urinary tract infection E. Roseola

D. UTI, the most common bacterial illness in a female infant, is consistent with her high fever, fussiness, and decreased appetite. Her CBC suggests that she has a bacterial infection (leukocytosis and elevated bands). A sample of her urine should be obtained by catheterization and sent for urinalysis and culture.

Case 18: A 6-week-old infant presents to your office for a check-up. The baby was born at term by NSVD to a 29-year-old G1P0 mother with no complications. Mother states the baby was feeding well until a week ago, when he developed increased sleepiness, prolonged feeding, and greater duration between feeds. His mother notes he stops to take breaks sometimes because he seems to be trying to catch his breath. He has 4 to 6 wet diapers per day and poopy diapers 3 or 4 times per day. Vital signs are: T: 37.6 C, RR: 68 bpm, P: 138 bpm, BP: 88/58 mmHg, and 02 saturation is 98%. The physical examination is notable for increased respiratory effort and retractions, and, upon cardiac examination, a murmur with a hyperactive precordium and no cyanosis. Abdominal exam reveals a liver edge palpable to 4 cm below the right costal margin. Which condition would be least likely to be the cause of the infant's symptoms? A. Aortic stenosis B. Coarctation of the aorta C. Ventricular septal defect D. Patent ductus arteriosus E. Atrial septal defect

E. Choice E is correct because atrial septal defects (ASDs) do not cause CHF. ASDs often go undiagnosed for decades due to subtle physical examination findings and/or a lack of appreciable symptoms. Children with ASD's are generally asymptomatic.

Case 25: A 5-month-old infant presents to urgent care with his mother who states that she witnessed her son stop breathing and turn blue for about 25 seconds. Upon physical stimulation by the mother, the patient began to breathe again. This is the first time she has ever witnessed this happening. The patient's birth and past medical history are unremarkable. Physical exam is unremarkable, vital signs are stable and normal, and lab studies are all within normal limits. Which of the following is LEAST likely to be on the differential diagnosis as a cause for this infant's BRUE (brief resolved unexplained event)? The best option is indicated below. Your selections are indicated by the shaded boxes. A. Seizures B. Arrhythmia C. Infection D. Gastroesophageal reflux E. Congenital heart disease

E. Choice E is correct because it is unlikely for a patient with congenital heart disease to first present with an BRUE. This patient has no past medical history and his birth history was unremarkable. Typically the patient will suffer from acute decompensation within the first few weeks of life. These patients will also have growth problems, difficulty with feeding and a murmur is often appreciated on physical exam. Children with undiagnosed Tetraology of Fallot may have intermittent episodes of cyanosis while blood supply is diverted from the pulmonary vasculature.

Case 12: Joe, a previously healthy 11-month-old infant with 5-day history of a "cold," is brought to the ED by mom for one day of acute worsening cough and intermittent wheezing. Per mom, the cough was initially dry but has become more "phlegmy," making it difficult for Joe to breathe, particularly when he is feeding or more active. His immunizations are up to date, and he has no known allergies. His family history is significant for a 6-year old sister who was diagnosed with asthma four years ago. On exam, Joe is afebrile, mildly tachypneic with normal O2 saturation. He has prominent nasal flaring and mild subcostal retractions. He has clear rhinorrhea but no evidence of oropharyngeal erythema. Lung exam reveals decreased breath sounds and wheezes on the right. What is the most likely diagnosis? A. RSV bronchiolitis B. Epiglottitis C. Viral URI D. Asthma E. Foreign body aspiration

E. Given Joe's age, foreign body aspiration should always be included in the differential diagnosis for acute onset wheezing. The lung findings of asymmetric breath sounds and wheezing support this diagnosis. Foreign body in the airway can be confirmed by bilateral decubitus or inspiratory/expiratory chest films, characterized by decreased deflation on the affected side. If complete obstruction, x-ray will generally reveal atelectasis (whiting out) and signs of volume loss (mediastinal shift towards affected side to compensate for loss of volume).

Case 15: A 15-month-old boy presents to the ED in January with a 3-day history of diarrhea. His current weight is 11 kg. He was born at 39 weeks, without any perinatal complications. There is no significant history of travel, sick contacts, or recent changes in diet. The mother notes that he has had only 2 diaper changes over the last day. Physical exam is remarkable for an irritable but consolable infant with tachycardia and normal blood pressure. He is crying without tears and his mucous membranes are dry. His abdominal exam is benign. There is no tenting, and capillary refill is 2 seconds. He is diagnosed with gastroenteritis and started on rehydration therapy. Which of the following statements is true? A. The patient is mildly dehydrated and should be managed with oral rehydration (Pedialyte). B. The patient is moderately dehydrated and should be managed with oral rehydration (Gatorade). C. The patient should be rehydrated with clear liquids and then transitioned to a lactose-free diet until his diarrhea resolves. D. The patient is moderately dehydrated and should be bolused with 220 ccs of D5 ½ normal saline for emergency phase correction, to ensure hemodynamic stability. E. ) The work-up for infectious diarrhea for this patient should include a Wright's stain for fecal WBCs, a stool Rotazyme, and a stool sample for culture and sensitivity.

E. In addition to correcting this patient's hydration status, a work-up for the infectious causes of this patient's diarrhea might include a stool Wright's stain for fecal WBCs (which would suggest a bacterial cause if this is infectious diarrhea), a Rotazyme test (given the high incidence of rotavirus in the winter months), and a stool sample for culture and sensitivity. Additional studies might include stool guaiac (for occult blood) and a check for stool C. diff toxin.

Case 26: A 4-week-old infant presents to clinic for a well child check. This infant had an uneventful delivery by NSVD at full term and subsequent normal neonatal screen. The nurse reports that her growth is a concern, with weight at 3.0 kg (< 3rd percentile) and weight for height at < 3rd percentile. Mom denies any drinking or drugs since before this pregnancy and says she has been breastfeeding every two to three hours and supplementing with appropriately mixed formula one to two times a day. She does report the baby seems to have issues latching and some possible gasping between suckles. There has been no diarrhea, hematochezia, vomiting, or fevers. The vital signs and exam (apart from a thin infant) are normal. The mother's affect is flat, and she seems anxious when you ask her about her infant. What is the most likely diagnosis for this infant's failure to thrive? The best option is indicated below. Your selections are indicated by the shaded boxes. A. Malabsorption B. Gastroenteritis C. Milk protein allergy D. Congestive heart failure E. Failure to thrive due to inadequate caloric intake

E. This infant is likely not consuming adequate calories to grow. The mother or primary caregiver may neglect proper feeding of the infant because of preoccupation with the demands or care of others, her own emotional problems, substance abuse, lack of knowledge about proper feeding, or lack of understanding of the infant's needs. It is also important to assess the mother for post-partum depression. Standardized screening tools, such as the Edinburgh, are used routinely in pediatric office settings for this purpose.

Case 11: A 3-year old girl comes to the clinic with a chief complaint of fever (104F) for over a week. Her mom reports that she has been fussy and inconsolable since she became febrile. She has a red tongue, with large papillae, conjunctivitis, a palmar rash, unilateral cervical adenopathy, as well as swollen feet. Given the most likely diagnosis, what is the most important follow-up for this girl over the next few weeks? A. Ophthalmology follow-up to determine extent of eye damage and determine need for corticosteroids B. Physical therapy follow-up to help prevent long-term joint deformities and ensure long-term functionality C. Cardiology follow-up to rule out presence of rheumatic fever D. Echocardiogram to look for coronary artery aneurysm E. Neurology follow-up to evaluate partial paralysis of lower extremities

The corect answer is D. A. Choice A is incorrect because the patient likely has Kawasaki disease. Early referral to an ophthalmologist is important for patients diagnosed with other conditions, including Stevens Johnson Syndrome (SJS) to determine the degree of eye involvement and if treatment with topical steroids is needed. SJS is a mucocutaneous disorder defined by fever, severe stomatitis (inflammation of the mucous lining of any of the structures in the mouth), conjunctivitis, and a blistering rash. It is typically caused precipitated by medications or infections. Early referral to an ophthalmologist would also be important for patients diagnosed with juvenile idiopathic arthritis, because they can suffer from uveitis, which if left untreated, can lead to long-term problems such as cataracts, glaucoma, or blindness. B. Choice B is incorrect because the patient in this case has Kawasaki disease. Early referral to physical therapy is important for patients diagnosed with systemic juvenile idiopathic arthritis, which is characterized by prolonged or spiking fever, rash, and arthritis. Early treatment and physical therapy can help prevent joint deformities and improve long-term functionality. C. Choice C is incorrect because the patient in this case has Kawasaki disease. Patients who have scarlet fever could develop rheumatic fever or post-streptococcal glomerulonephritis, among other problems. Scarlet fever is characterized by a "erythematous, blanching, sandpaper-like rash" with very fine papules secondary to infection with Group A streptococcus. It may start in the groin, axilla, or neck, before spreading rapidly over the trunk and extremities. Fever can be high, but generally resolves within five days. D. Choice D is correct because children with Kawasaki disease are at high risk for coronary artery aneurysm formation and should receive an echocardiogram within four weeks of the onset of their illness. Use of IVIG for the treatment of Kawasaki disease has decreased the risk of coronary artery aneurysms significantly. Kawasaki disease is diagnosed when there is a fever plus four of the following: changes in oral mucosa (e.g., strawberry tongue), extremity swelling or redness, unilateral cervical adenopathy, conjunctivitis, and rash. Infectious and rheumatologic causes must be excluded in order to make the diagnosis of Kawasaki disease. E. Choice E is incorrect because the patient in this case has Kawasaki disease. Neurologic issues such as paralysis of a lower limb can occur in severe cases of Rocky Mountain Spotted Fever, which is characterized by fever, myalgias, headache, and petechial rash classically starting on wrists and ankles and progressing centrally.

Case 8: A 3-week-old infant is brought to his pediatrician with a chief complaint of light tan-colored stools and worsening jaundice. He is exclusively breastfed and has 6-8 wet diapers per day. On exam, he appears to have scleral icterus and jaundice. Upon further workup, he is found to have an elevated direct bilirubin. What is his most likely diagnosis? A. Biliary atresia B. Breastfeeding jaundice C. G6PD deficiency D. Physiologic jaundice E. Caput succedaneum

The correct answer is A. A. Biliary atresia can present anytime between birth and 8 weeks of age, but usually occurs after 2 weeks of age. Jaundice is usually the first presenting finding, along with acholic stools, dark urine (from increased bilirubin excretion) and hepatosplenomegaly if the problem goes unrecognized. Laboratory values classically show an increased level of direct or conjugated bilirubin > 2 mg/dL. If biliary atresia is confirmed with further laboratory testing and imaging, surgical intervention must be pursued as soon as possible.

Case 1: A 19-year-old female in her 38th week of pregnancy goes into active labor. Shortly after birth her baby is noted to have a high-pitched cry, tremulousness, hypertonicity, and feeding difficulties. The baby is otherwise developmentally normal and the remainder of the physical exam also is normal. What is the drug the baby's mother likely used during her pregnancy? A. Heroin B. Alcohol C. Marijuana D. Cocaine E. Tobacco

The correct answer is A. A. Heroin is the correct choice. Opiate use during pregnancy may result in several different symptoms, including CNS findings (irritability, hyperactivity, hypertonicity, incessant high-pitched cry, tremors, seizures), GI symptoms (vomiting, diarrhea, weight loss, poor feeding, incessant hunger, excessive salivation), and respiratory findings (including nasal stuffiness, sneezing, and yawning). B. Alcohol is incorrect. Fetal alcohol syndrome has a distinct pattern of facial abnormalities, growth deficiency, and CNS dysfunction. These infants may also exhibit other neurobehavioral deficits such as poor motor skills and hand-eye coordination and learning problems, such as difficulties with memory, attention, and judgment. C. Marijuana is incorrect. There is limited evidence for a withdrawal syndrome associated with marijuana use. D. Cocaine use during pregnancy is not typically associated with withdrawal symptoms. Cocaine has been linked to subtle deficits appreciated later in childhood, including deficits in cognitive performance, information processing, and attention to tasks. E. Tobacco is incorrect. Smoking is not associated with the withdrawal syndrome described above. Smoking has been linked in a dose-dependent manner with lower weight newborns at birth. There is a two-fold increase in low birth weight even in light smokers (< 10 cigarettes per day). Smoking during pregnancy also has been associated with subtle neurodevelopmental deficits in some exposed children.

Case 7: A 3-hour-old infant, born by C-section at 36 weeks to a 30-year-old G1P1 with Apgars of 8 and 9 at 1 and 5 minutes, respectively, is found to be tachypneic in the newborn nursery. His mother has a history of Type II diabetes that was poorly controlled during her pregnancy. She was compliant with prenatal vitamins and took no other drugs during her pregnancy. Prenatal labs, including GBS, were negative. The mother's membranes ruptured 9 hours prior to delivery, she was afebrile, and the amniotic fluid had no meconium. On physical exam, the infant is large for gestational age. He has good air movement through the lungs bilaterally, without retractions or nasal flaring. He appears well perfused with normal cardiac exam. He has decreased muscle tone and a weak suck reflex. A screening test at 3 hours of life reveals blood glucose of 39 mg/dL. What is the most likely diagnosis? A. Hypoglycemia B. Transposition of the great arteries C. Transient tachypnea of the newborn D. Neonatal sepsis E. Pneumothorax

The correct answer is A. A. Hypoglycemia is a common presentation in an infant born to a diabetic mother with poor glucose control during her pregnancy. The increase in maternal serum glucose stimulates fetal pancreatic beta cells to increase insulin production, and this hyperinsulinemic state leads to hypoglycemia when the placental glucose supply is discontinued after delivery. At < 4 hours of life, a glucometer reading of < 25 mg/dL without symptoms or < 40 mg/dL with symptoms would require intervention to correct the hypoglycemic state. This infant has signs of hypotonia, with absence of flexed posture and weak suck, and a blood glucose reading of 39 mg/dL, making hypoglycemia the most likely diagnosis. B. Transposition of the great arteries is a congenital heart defect in which the aorta and pulmonary artery are switched, resulting in poorly oxygenated blood pumped into the systemic circulation. The infant with transposition is generally cyanotic and will be in respiratory distress. This congenital defect is usual accompanied by a VSD, and maternal diabetes is a risk factor. This infant appears well perfused on exam, and has no murmurs on cardiac exam, making transposition a less likely diagnosis. C. Transient tachypnea of the newborn (TTN) is a condition characterized by delayed clearance of amniotic fluid from the infant's lung following birth (persistent postnatal pulmonary edema) resulting in respiratory distress. Infants born by C-section and to diabetic mothers are at an increased risk of TTN. X-ray findings include "wet" appearing lungs with significant perihilar streaking, interstitial and alveolar fluid, and fluid in the pleural space and along the fissures. TTN generally resolves within 24 to 48 hours and is treated symptomatically. D. Neonatal sepsis is most commonly caused by GBS, Listeria and E. coli, transmitted from mother to baby. Additional risk factors include premature rupture of membranes (> 18 hours prior to delivery), preterm delivery, and chorioamnionitis. Infants may present with fever, trouble breathing, jaundice, and lethargy. Our infant is premature and tachypneic, but he is afebrile with normal Apgars and no evidence of altered level of alertness. Furthermore, mother was GBS negative, afebrile (no chorioamnionitis), with no premature rupture of membranes, making this diagnosis less likely. E. Pneumothorax is collapse of lung tissue secondary to air accumulation in the pleural space. Risk factors for pneumothorax in an infant include previous intubation or underlying lung disease (such as severe respiratory distress syndrome). Characteristic physical exam findings include asymmetric breath sounds or decrease in breath sounds on one side. This infant has good air movement in bilateral lung fields, making this diagnosis less likely.

Case 10: A 3-month-old male presents to the ED with a fever that started the previous day. Mother reports that he was fussy and had decreased oral intake. He had had five fewer diaper changes than usual. He had no vomiting, diarrhea, or respiratory difficulty. On physical exam his temperature is 101.6 F, pulse 110 bpm, RR 24 bpm, and BP 95/67 mmHg. The baby seems irritable and is not consolable by the parent. HEENT exam was significant for dry mucous membranes. Other than his irritability, the rest of the physical exam was unremarkable. CBC showed WBC 3.5, but was otherwise normal. BMP was within normal limits. Urinalysis showed positive leukocyte esterase, positive nitrite, and WBCs > 10/hpf. An LP was performed, and urine and CSF culture results are pending. The patient is placed on IV fluids and is started on cefotaxime. What is the next best step in evaluation? A. Renal bladder ultrasound B. Kidney-ureter-bladder (KUB) x-ray C. Intravenous pyelogram D. VCUG E. Oral ampicillin

The correct answer is A. A. This infant has a fever without other respiratory symptoms. Meningitis and UTI must be considered in patients with fever. The only way to rule out meningitis is by lumbar puncture. This patient has a low WBC, suspicious for sepsis, and a UA that is highly suggestive of UTI. Empiric therapy should be started to cover common organisms including E.coli, P. mirabilis, and Klebsiella. Cefotaxime is reasonable empiric therapy. Renal ultrasound is recommended for all infants with pyelonephritis to assess for renal structural abnormalities or signs of obstructive uropathy (hydronephrosis). B. KUB is not recommended for UTI. C. Intravenous pyelogram would expose the patient to radiation and would not be recommended to screen for renal abnormalities. D. VCUG screening is recommended only for recurrent UTI or when there is abnormal renal ultrasound. E. The patient is already on parenteral antibiotics, so oral antibiotics would not be necessary. Also, ampicillin would not provide empiric coverage.

Case 1: A mother brings her 20-day-old male infant to your clinic for the child's first visit. You learn that the infant was born at home to a 28-year-old G1P1, and the infant has not yet received newborn screening. During your history, you learn that the infant has been vomiting 2 to 3 times per day, and the mother reports that her son seems fussier than her friends' infants. On exam, you note an eczematous rash and a musty odor to the infant's skin and urine. Which enzyme deficiency would you expect the infant to display? A. Phenylalanine hydroxylase B. Cystathionine synthase C. Sphingomyelinase D. Alpha-L-iduronidase E. Glucose-6-phosphatase

The correct answer is A. A. This infant likely has phenylketonuria (PKU), an autosomal recessive disorder of amino acid metabolism caused by a deficiency in the enzyme phenylalanine hydroxylase. Affected infants are normally detected by newborn screening, but can present with vomiting, hypotonia, musty odor, developmental delay, and decreased pigmentation of the hair and eyes. The best developmental outcomes occur if a phenylalanine-restricted diet is initiated in infancy.

Case 7: A male infant weighing 3200 grams is born to a G1P1 female at 39 weeks' gestational age via planned C-section. Maternal PMH is unremarkable, and GBS status is unknown. Apgars are 7 and 8 at 1 and 5 minutes of life, respectively. The delivery is uncomplicated, and the infant initially appeared in good condition. However, one hour following delivery the infant develops increasing respiratory distress. Respiratory rate is assessed as 90 breaths/min. All other vital signs are within normal limits. On exam, the infant is acyanotic with rapid respirations and robust capillary refill. Chest x-ray shows bilateral lung fields with the appearance of "a radio-opaque line of fluid in the horizontal fissure of the right lung." No air bronchograms are noted. What is the most likely etiology of the infant's respiratory distress? A. Transient tachypnea of the newborn (TTN) B. Respiratory distress syndrome (RDS) C. Neonatal sepsis D. Meconium aspiration

The correct answer is A. A. Transient tachypnea of the newborn (TTN) is the most likely underlying etiology. This condition is caused by residual fluid in the infant's lungs following delivery, and usually resolves within several days. It is more common in babies delivered via C-section, as the normal mechanical force of labor that helps expel fluid from the lungs is lacking. Babies with TTN and other forms of respiratory distress are often unable to nurse and require feeding via NG tube until respiratory status stabilizes.

Case 10: A 6-month-old female with normal birth and developmental history presents with fever for the past two days, fussiness, and decreased appetite. ROS is negative. No abnormalities are noted on the physical examination. A urinalysis from a bag specimen is positive for leukocytes and nitrite, which suggests the presence of a UTI; a culture from this sample is pending. The patient is ill-appearing, dehydrated, and unable to retain oral intake. She is hospitalized, receives a 20 cc/kg NS bolus and is placed on maintenance IV fluids with clinical improvement. What is the best next step for management of this patient? A. Urinary catheterization B. Renal bladder ultrasound C. Begin parenteral antimicrobials D. Midstream clean catch urine collection E. Increase intravenous fluid administration rate to flush the kidneys

The correct answer is A. A. Urinary catheterization is correct . It is the best method for obtaining a specimen for culture that has not been contaminated by perineal bacteria, and for this ill child, you must determine the cause of the fever with accuracy. B. Renal ultrasound may be indicated if the infant is found to have a urinary tract infection but is not indicated as part of the initial work up. C. With a bag culture pending, you may have a contaminated/confusing culture result and may have to rely solely on clinical criteria to treat this presumed UTI. D. Midstream clean catch specimen is incorrect because it is not ideal in a patient who has not been toilet-trained and cannot void on demand. E. While it is important to provide hydration, this patient has responded well to the initial fluid administration, and there is no evidence that increasing this beyond normal recommendations is helpful.

Case 16: A 7-year-old boy is brought by ambulance to the ED because of decreased mental status for 2 hours. His parents say that he has been tired for the past 3 days. Six hours ago, he began vomiting, but has not had diarrhea. He is difficult to arouse and moans with stimulus. Temperature is 98.6, pulse is 124 beats/minute, respirations are 28 breaths/minute, blood pressure is 96/68 and oxygen saturations are 99% on room air. Physical exam findings include dry mucous membranes and moaning on palpation of the abdomen. The remainder of the exam is normal. What is the most likely cause of his condition? A. Appendicitis B. Diabetic ketoacidosis C. Opioid overdose D. Non-accidental trauma E. Sepsis

The correct answer is B. A. Appendicitis would rarely present with altered consciousness. The usual history with appendicitis is onset of periumbilical pain that persists over hours, migrating to the right lower quadrant. Vomiting could be present, and tachycardia may be present due to pain or dehydration, but altered mental status would be unusual. On physical exam, peritoneal signs may be present as well as psoas, obturator, or Rovsing's sign. B. DKA typically presents with altered mentation, vomiting, dehydration, and abdominal pain. The history will yield polydipsia and polyuria during the days preceding DKA. Metabolic acidosis causes tachypnea as the body tries to blow off CO2 through a compensatory respiratory alkalosis. C. Although he may have ingested opioids, his presentation does not fit well with this choice. Signs of opioid overdose include pinpoint pupils, depressed respiratory rate, and altered consciousness. His tachypnea and lack of pinpoint pupils argue against this choice. D. Lack of fractures, bruises, or burns argues against this choice. Trauma resulting in increased intracranial pressure may result in hypertension, bradycardia, and disordered breathing. E. Sepsis can present with altered mental status. This child's presentation is less consistent with sepsis given that he doesn't have fever or other vital sign changes consistent with sepsis syndrome (temperature > 38.5°C or < 36°C, hypotension, along with warm, dry extremities).

Case 15: You are seeing a 1-month-old infant who is < 3rd percentile for weight. He is breastfed every 2 hours and latches on well. However, he has frequent non-bilious episodes of vomiting that have been increasing over the past week despite his mother taking "reflux precautions." He does not have mucus or blood in his stool. Physical exam reveals a small, olive-sized mass in his abdomen. What is the most likely diagnosis? A. Cleft palate B. Pyloric stenosis C. Cystic fibrosis D. Non-organic failure to thrive E. Munchausen syndrome by proxy

The correct answer is B. A. Choice A is incorrect. Children with structural anomalies of the palate typically present with difficulty feeding. This child appears to be latching on well, but has difficulty retaining the food once ingested. B. Choice B is correct because the history of frequent vomiting, poor weight gain, and the finding of an abdominal mass are consistent with pyloric stenosis. Children with pyloric stenosis often present at 3 weeks of age. C. Choice C is incorrect. Children with cystic fibrosis typically present with failure to thrive secondary to chronic malabsorption, with characteristic loose and malodorous stool. Lab testing reveals elevated sweat chloride. There may be a known family history. D. Choice D is incorrect. Several findings in this patient point toward an organic rather than an inorganic cause of failure to thrive, and organic causes such as pyloric stenosis should be ruled out before an inorganic cause is assumed. E. Choice E is incorrect. Although maltreatment of a child should always be considered, particularly in the youngest children. This child has sufficient findings by history and physical exam to suspect true organic etiology.

A 9-year-old boy is brought to the ED in a coma secondary to diabetic ketoacidosis. Which of the following laboratory results would NOT likely be found in this child? A. Anion gap of 20 mEq/L B. Potassium of 3.3 mEq/L C. Venous pH of 7.1 D. Sodium of 132 mEq/L E. Creatinine of 1.0 mEq/L

The correct answer is B. A. DKA causes a metabolic acidosis from the elevated level of ketones. The elevated level of ketoacids and lactic acid requires buffering by bicarbonate, thus leading to an increased anion gap. B. In diabetic ketoacidosis, the acidosis and lack of insulin cause potassium to leave cells and enter the serum, causing an elevated serum potassium level. However, as the DKA is corrected and insulin is administered, the potassium will re-enter the cells, causing a decreased serum potassium level, so potassium levels should be monitored closely when therapy is initiated. C. The pH would be low due to the metabolic acidosis caused by the elevated level of ketones. D. Hyponatremia is seen in DKA because the hyperosmolarity of the intravascular space from the increased glucose levels causes osmotic movement into the extracellular space. Additionally, there is increased sodium loss from the kidneys. A corrected sodium level should be calculated to adjust for the hyperglycemia, using the following formula: corrected sodium = [{(measured glucose - 100)/100} x 1.6] + measured sodium. E. Patients with DKA are often dehydrated when they present to the ED. This causes a prerenal azotemia, which presents as an elevated creatinine level.

Case 1: A 33-year-old G1P0 female with a history of medically controlled seizures gives birth vaginally to a boy with IUGR at 38 weeks' gestation. The newborn is noted to have dysmorphic cranial features and his head circumference is 28.5 cm (< 5th percentile). What is another associated abnormality you might expect to see in this newborn? A. Hepatosplenomegaly B. Cardiac defects C. Absent red reflex D. Chorioretinitis E. Tremors

The correct answer is B. A. Hepatosplenomegaly in newborns is seen in metabolic diseases, storage diseases, HIV vertical transmission, intrinsic liver disease, and in congenital infections. Maternal anticonvulsant use does not cause hepatosplenomegaly. B. The mother was on an anticonvulsant for her seizures. Taking anticonvulsants during pregnancy may lead to cardiac defects, dysmorphic craniofacial features, hypoplastic nails and distal phalanges, IUGR, and microcephaly. Mental retardation may be seen. A rare neonatal side effect is methemoglobinuria. C. The red reflex is the normal reddish-orange reflection of light from the eye's retina that is observed when using an ophthalmoscope. An absent red reflex in a newborn can be due to congenital cataracts or a retinoblastoma. D. Chorioretinitis in a newborn may be due to congenital toxoplasmosis and CMV infections. It is not caused by maternal anticonvulsant use. E. Tremors in a newborn can be due to maternal substance use. They would not result from maternal use of anticonvulsants.

Case 15: Rashid is a 5-week-old infant who presents to clinic with 4 days of repeated, forceful, non-bilious, non-bloody vomiting without diarrhea. He has 8 to 9 episodes of vomiting per day immediately following breastfeeding. The episodes started 2 weeks after the entire family suffered from severe viral gastroenteritis. His birth history is uncomplicated (full term, NSVD, unremarkable 30-week ultrasound) and birth weight was 3.6 kg (50th percentile). On exam, his vitals are: T 36.7°C, HR 185, BP 85/45, RR 36, Wt 4.1 kg (25th percentile). On exam, his eyes are moderately sunken without production of tears, his lips are cracked, and his throat is without erythema. His capillary refill is ~3 seconds, and his pulse is thready. What is your first step in management? A. Close observation in the office for 6 hours and encourage PO intake until vitals normalize. B. Intravenous lactated Ringer's solution of 20mL/kg boluses until baseline clinical status is achieved, then 100 mL/kg oral rehydration solutions over next 4 hours. C. Intravenous 20 mL/kg boluses of ¼ normal saline solution until baseline clinical status is achieved, then closely monitor vitals for 6 hours while encouraging PO formula intake. D. Observe for 6 hours with normal PO intake and administer 60-120 mL of oral rehydration solution for every episode of vomiting. E. Administer 75 mL/kg of oral rehydration solution over 3-4 hours and 60-120 mL of oral rehydration solution for every episode of vomiting

The correct answer is B. A. Observing closely in the office for 6 hours and encouraging PO intake until vitals normalize are not appropriate measures for treating this child's severe dehydration. Signs of severe dehydration include lethargy or unconsciousness on exam, poor PO intake, tachycardia, weak or nonpalpable pulses, deep breathing, deeply sunken eyes, parched mouth and tongue, reduced skin turgor, and cold/cyanotic extremities. In cases like this, the child must be placed on immediate IV fluids with 20 mL/kg boluses until vitals and mental status normalize. B. Lactated Ringer's solution or normal saline in 20 mL/kg boluses until urine output is established and mental status improves, then 100 mL/kg oral rehydration solutions over next 4 hours. This follows current CDC guidelines for treating a severely dehydrated child. Intravenous hydration with 5% dextrose ½ normal saline at twice maintenance fluid rates may be substituted for the oral rehydration solution if the child is not tolerating PO intake. To replace ongoing losses, the CDC recommends 60-120mL of oral rehydration solution per diarrheal/emetic episode (through a nasogastric tube, if necessary). C. One quarter normal saline (1/4 NS) is a hypotonic solution, and would not be ideal for the treatment of dehydration due to emesis. The recommended therapy to correct severe dehydration is to give 20 mL/kg boluses of isotonic solution and to reassess for clinical improvement following each administration. Once the patient is stable and back to baseline, then continue IV hydration with 5% dextrose ½ normal saline at twice maintenance fluid rates OR give 100 mL/kg oral rehydration solution over 4 hours. D. This would not be a recommended treatment for the severely dehydrated child since it relies on normal PO intake. However, the replacement of losses strategy is correct for all patients < 10 kgs. E. IV rehydration is key to initial fluid resuscitation in the severely dehydrated, so this treatment would not be sufficient. However, this plan would be appropriate for mild to moderately dehydrated patients.

Case 7: A 30-minute-old infant is evaluated in the nursery for tachypnea. He was born at 30 5/7 weeks gestation by vaginal delivery. Labor was induced due to severe maternal pre-eclampsia. Mother received no prenatal care. Apgars were 6 and 7 at 1 and 5 minutes, respectively. One dose of betamethasone was given prior to delivery. A chest x-ray reveals decreased lung expansion. Which of the following diagnoses is most consistent with this patient's presentation? A. Meconium aspiration syndrome (MAS) B. Respiratory distress syndrome (RDS) C. Persistent pulmonary hypertension (PPHN) D. Transient tachypnea of the newborn (TTN) E. Bronchopulmonary dysplasia (BPD)

The correct answer is B. A. This choice is incorrect. Although the presence or absence of meconium was not noted in the case, it is known that from 20 to 34 weeks' gestation, the fetus will pass meconium infrequently. Most cases of MAS are in term or post-term infants. On chest x-ray, we might see overdistention of the lung or other sequelae, such as pneumothorax. B. This choice is correct. The baby boy is preterm, and his mother received only one dose of betamethasone, which puts him at increased risk for developing infant RDS, which is caused by insufficient surfactant. His physical exam and chest x-ray findings are consistent with RDS. A reticulogranular or "ground-glass" appearance and bronchograms may be present shortly after birth, but take 12-24 hours to maximally develop if surfactant is not administered. C. This choice is incorrect. PPHN generally occurs in babies born after 34 weeks. There are several causes for PPHN: underdevelopment, maldevelopment, and maladaptation. Underdevelopment of the lungs can be secondary to congenital diaphragmatic hernia, oligohydramnios in utero, IUGR, or renal agenesis. The underdevelopment causes increased pulmonary vascular resistance and has a poor prognosis. Maldevelopment involves remodeling of pulmonary vasculature and is associated with post-term delivery and meconium aspiration syndrome. Maladaptation can be caused by infection with GBS. Vasoactive mediators are activated by bacterial phospholipids, causing an increase in pulmonary vascular resistance. D. This choice is incorrect. TTN is a disorder of delayed reabsorption of fluid in the newborn's lungs. Prematurity, delivery by C-section, being large or small for gestational age, or having a diabetic mother are all risks. In order to be diagnosed with TTN, the baby would need to show improvement within several hours. Although this is on the differential for the newborn baby's condition based on clinical presentation, a chest x-ray should have shown perihilar streaking and other evidence of interstitial fluid. E. This choice is incorrect, because BPD is the result of prolonged mechanical ventilation. Our patient is at risk for developing this syndrome if he requires intubation. Chest x-ray may show atelectasis, inflammation, or pulmonary edema. With severe disease, the chest x-ray may reveal fibrosis and hyperinflation.

Case 1: A 19-year-old G1P0 presents in labor to the ED at 38 gestational weeks. On interview it is discovered that the patient had irregular prenatal care, drank a couple of beers every weekend, and smoked 4 cigarettes a day. She delivers a baby boy who is small for gestational age. On exam, it is noted the baby has microcephaly, a smooth philtrum, and a thin upper lip. What do you suspect caused these features in the baby? A. Tobacco exposure B. Alcohol exposure C. Congenital rubella D. Vertically transmitted HIV E. Congenital CMV infection

The correct answer is B. A. This choice is incorrect. While tobacco exposure can cause infants to be small for gestational age they typically do not have any characteristic facial features. B. This choice is correct. Fetal alcohol syndrome has very characteristic facial features, including a smooth philtrum, thinning of the upper lip, and small palpebral fissures. C. This choice is incorrect. Congenital rubella presents with sensorineural deafness, eye abnormalities (retinopathy, cataracts), and patent ductus arteriosus. D. This choice is incorrect. Typically, vertically transmitted HIV does not lead to recognizable symptoms at birth. This diagnosis cannot be completely ruled out without lab testing. E. This choice is incorrect. Symptomatic congenital CMV infection presents with microcephaly, jaundice, hepatosplenomegaly, low birth weight, and petechiae at birth.

Case 8: A 4-day-old infant presents for his first pediatric well-child visit. His birth history consists of an uncomplicated normal spontaneous vaginal delivery after 7 hours of labor-no vacuum or forceps assistance were used. The patient is the first child to a 30-year-old mother of Mediterranean descent. Mom is very concerned that her baby has started to look "yellow" since leaving the hospital. She has been breastfeeding every 2-3 hours and says that the baby latches on for 1-5 minutes for each feed. He has had few wet diapers, and mom is concerned he is not getting enough to eat. Which of the following would most aid in narrowing the differential diagnoses? A. Newborn screen results B. Fractionated bilirubin C. WBC D. Blood smear E. No further workup is needed, as this is likely physiologic jaundice

The correct answer is B. B. The test that will give you the most information at this juncture is a fractionated bilirubin. With the knowledge of the total serum bilirubin (TSB) and direct serum bilirubin, one will be able not only to narrow the differential (hemolysis vs. obstruction), but also to guide treatment (i.e., indirect serum bilirubin may be above phototherapy level). TSB can also indicate if the situation requires more drastic measures, such as a transfusion exchange.

Case 16: A 9-year-old female is brought to clinic by her mother because of two days of abdominal pain and vomiting. She has vomited six times today and has had decreased appetite, but no diarrhea, fevers, sick contacts, or changes in diet. Her mom states that she has been otherwise healthy apart from increased thirst and occasional bedwetting over the last few weeks. Of note, patient's maternal grandmother suffers from celiac disease. On exam, patient is afebrile and has a HR of 180 bpm, BP 90/60 mmHg, RR 50 bpm, and O2 saturation of 98%. She is lying in bed appearing slightly drowsy, taking rapid, deep breaths and is slow to respond to questions. Her heart and lung exams are normal apart from being tachycardic, and abdominal exam reveals mild diffuse tenderness to palpation with no rebound or guarding. Which of the following would be the most appropriate next step in management? A. AP and lateral chest x-ray B. Midstream urine culture C. Fingerstick blood glucose D. Abdominal ultrasound E. Gastric lavage

The correct answer is C. A. A chest x-ray would be appropriate if bacterial pneumonia were high on your differential. A patient with pneumonia generally presents with fever, cough, tachypnea, and will likely have characteristic lung findings such as crackles on exam. The patient may experience abdominal pain secondary to pleural inflammation; however, vomiting is not a common presentation. The absence of fever, cough, sick contacts, and lung findings on exam make pneumonia a less likely diagnosis. B. Urine culture would be appropriate if pyelonephritis were high on your differential. A patient with pyelonephritis may present with history of fever, dysuria, urinary frequency, CVA tenderness, and vomiting. However, this patient's overall clinical picture does not support the diagnosis, since the patient is afebrile without a history of dysuria or classic CVA tenderness. C. Obtaining a fingerstick blood glucose is the diagnostic step with the highest yield since the patient's clinical picture is strongly indicative of diabetic ketoacidosis (DKA). DKA is a condition more closely associated with type 1 (rather than type 2) diabetes, and is formally diagnosed if a random glucose is > 200 mg/dL, venous pH is < 7.3 and/or bicarbonate < 15 mEq/L and there is ketonemia or ketonuria. Patients in DKA can present with abdominal pain and vomiting secondary to metabolic acidosis that stems from ketonemia and lactic acidosis. Furthermore, osmotic diuresis from hyperglycemia may contribute to dehydration, which can manifest as tachycardia, hypotension, and altered mental status. In an attempt to compensate for the metabolic acidosis, the patient may also present as tachypneic with characteristic Kussmaul respirations (rapid, deep breaths). This patient's history of polydipsia, enuresis, and family history of autoimmune disease (including celiac disease and Hashimoto's thyroiditis) suggest that the patient has type 1 diabetes. Her current vital signs and general state of lethargy also point towards DKA and should be confirmed with a fingerstick glucose (in addition to other tests). D. Abdominal ultrasound would be indicated if appendicitis were high on your differential. Appendicitis may present with fever, nausea, vomiting, diarrhea, decreased appetite, and abdominal pain that localizes to the RLQ. Physical exam findings may include a positive Rovsing's sign or a positive psoas or obturator sign. Ultrasound may be a useful tool in detecting appendicitis in children. Appendicitis is less likely in this patient since she is afebrile and does not have the classic localizing pain characteristic of this condition. E. Gastric lavage may be the indicated next step of management if toxic ingestion is the confirmed diagnosis. A patient who has ingested toxic substance is usually afebrile and obtunded and may present with vomiting and manifestations of dehydration (secondary to vomiting). In a patient with salicylate toxicity, tachypnea is a common presentation. A thorough history to assess exposure to toxins and sending urine for evaluation of toxins may be appropriate to make this diagnosis. Given this patient's overall picture, DKA is a more likely diagnosis

Case 14: An 18-month-old presents with yellow and poorly mobile tympanic membranes. Four months prior he presented then with several days of nasal congestion, cough, decreased eating and ear tugging. His exam then revealed a red, nonmobile tympanic membrane and he was treated with amoxicillin. Based on the history and physical exam, what is the most likely diagnosis now? The best option is indicated below. Your selections are indicated by the shaded boxes. A. Mastoiditis B. Acute otitis media C. Otitis media with effusion D. Otitis externa E. Viral encephalitis

The correct answer is C. A. Although mastoiditis presents with tympanic membrane changes, the signs of bilateral yellow and poorly mobile tympanic membranes are more suggestive of otitis media with effusion. B. The course of development of this infection over four months makes this diagnosis unlikely. C. The earlier diagnosis of acute otitis media together with current findings of bilateral yellow and poorly mobile tympanic membranes on physical exam make this the most likely diagnosis. D. Although otitis externa presents with otalgia, the sings of bilateral yellow and poorly mobile tympanic membranes are more suggestive of otitis media with effusion. E. Viral encephalitis typically causes neurologic dysfunction with signs that include fever, headache, nausea, vomiting, photophobia, altered level of consciousness, seizures, and possibly focal neurologic signs. This child's presentation is much more consistent with otitis media with effusion.

Case 15: A 6-month-old infant comes to clinic because of several weeks of vomiting after large feedings. The vomiting has become blood-streaked, which is when the mom became concerned and brought him in. The baby's PO intake has been down and he has been losing weight. Abdominal exam is normal, with no masses palpated. What is the most likely diagnosis? The best option is indicated below. Your selections are indicated by the shaded boxes. A. Pyloric stenosis B. Gastroenteritis C. GERD D. Volvulus E. Intussusception

The correct answer is C. A. Choice A is incorrect because pyloric stenosis is characterized by a pattern of forceful, projectile, non-bilious vomiting, usually in younger infants. Infants are usually hungry and nurse avidly. An oval mass, 5-15 mm in longest dimension can be felt on deep palpation in the right upper abdomen, especially after vomiting. B. Choice B is incorrect. Large watery stools, which this patient did not have, are the hallmark of infectious gastroenteritis. Also, gastroenteritis is a more acute disease, while this patient is having more chronic symptoms. C. Choice C is correct because regurgitation/spitting up may be difficult to distinguish from true vomiting. Infants who reflux with overfeeding may sometimes have forceful vomiting. Severe esophagitis may result in blood-streaked emesis. Pain from reflux or esophagitis may lead to feeding aversion when gastroesophageal reflux is severe. D. Choice D is incorrect because in volvulus, blood may be seen in the stool but not typically in the vomitus. Bowel ischemia from volvulus can cause significant abdominal pain. E. Choice E is incorrect because in intussusception we might see the classic "currant jelly" stools. The abdominal exam in children with intussusception often shows the presence of a sausage-like mass due to the telescoped bowel.

Case 16: A 9-year-old boy presents to the ED in an ambulance after he was found unconscious at a local playground. In the ED he is arousable but difficult to arouse. He is able to minimally verbalize that his head and stomach hurt. He vomits clear liquid twice over the course of 30 minutes. Vital signs are as follows: T 37.6 C, P 66 bpm, BP 155/80 mm Hg, RR 18 bpm. His respirations are irregular with brief episodes of apnea. On physical exam you are unable to reproduce the abdominal pain and there is no rebound tenderness or guarding. The rest of the physical exam is unremarkable. Which of the following is the most likely diagnosis? A. Diabetic ketoacidosis B. Appendicitis C. Intracranial hemorrhage D. Viral gastroenteritis E. Small bowel obstruction

The correct answer is C. A. Choice A is incorrect, because in a patient with DKA one would expect increased adrenergic tone leading to tachycardia, not inappropriate slowing of the HR. Secondly, the patient's breathing pattern is more consistent with Cheyne-Stokes respirations, not Kussmaul breathing. Kussmaul breathing is typically characterized by deep breaths that may be rapid, normal or slow in rate without periods of apnea, often associated with metabolic acidosis. Lastly, one would expect signs or symptoms consistent with dehydration such as polyuria, polydipsia, decreased skin turgor, or skin tenting. However, the altered mental status, vomiting, headache, and abdominal pain could be seen in DKA. B. Choice B is incorrect because the lack of fever, inability to reproduce the abdominal pain in the RLQ, and the severely altered mental status argue against the diagnosis. In appendicitis, children will often complain of a migratory pain that beings around the periumbilical region and migrates to the RLQ. Patients often complain of rebound tenderness and demonstrate guarding as well. C. This is the correct choice. Increased ICP can be secondary to epidural or subdural hemorrhage. It is possible the patient may have fallen while playing in the playground. Increased ICP can present as the classic Cushing's triad: hypertension, inappropriate slowing of the heart rate, and irregular respirations (Cheyne-Stokes respiration). A further complication of increased ICP is epigastric discomfort. This is caused by the elevated ICP causing vagal stimulation, resulting in the secretion of gastric acid. Lastly, the patient's headache and non-bilious vomiting can also be ascribed to the increased ICP. D. Choice D is incorrect because gastroenteritis usually presents with fever, colicky abdominal pain, and diarrhea. It would also be atypical for the patient's mental status to be so adversely affected by gastroenteritis. More than 95% of gastroenteritis hospitalizations occur in children younger than 5 years, with the peak incidence between 3 and 24 months of age. The incidence tends to peak in winter. There can be both viral and bacterial causes for gastroenteritis. Classically, viral gastroenteritis will present with diarrhea in which the stool lacks blood or mucus. E. Choice E is incorrect because a small bowel obstruction usually presents with bilious vomiting, abdominal distention, inability to pass flatulence, and moderate-to-severe abdominal pain. The pain is often paroxysmal, coming and going in 4 to 5 minute intervals. The patient's lack of a fever, however, is consistent with a GI obstruction. The most common causes of a small bowel obstruction are adhesions from a previous surgery or a hernia.

Case 11: A 5-year-old girl, previously healthy, presents with an erythematous, vesicular rash on the palms and soles and a high fever for several days. Upon examination, she is also found to have ulcers in her mouth. A few days later, the fever and rash resolve. What is the most likely pathogen? A. Herpes simplex virus 1 (HSV-1) B. HIV C. Enterovirus D. Human herpesvirus 6 (HHV-6) E. Group A strep

The correct answer is C. A. HSV-1 causes gingivostomatitis and can sometimes be accompanied by fever and malaise, but lesions on the hands and feet would be uncommon. B. HIV infection can increase the risk of oral lesions secondary to infections by HSV-1 or Candida albicans, but again would be unlikely to cause lesions on the hands and feet. Furthermore, at this patient's age (5 years), she is unlikely to be HIV-positive unless it was vertically transmitted from her mother. C. This presentation is consistent with infection by cocksackie A, an enterovirus. Following an incubation period of three to five days, patients have fever, tender vesicles on their hands and feet, and oral ulcers. Sometimes the rash also occurs on the buttocks and the genitals. The infection resolves spontaneously within three days, and is spread from person to person via saliva, fluid from the vesicles, stool, or nasal discharge. D. HHV-6 is the virus that causes roseola, which manifests as fever followed by a macular or maculopapular rash, but this rash begins on the trunk, eventually spreading to the extremities, and does not cause oral lesions. E. Group A strep infection could cause fever and a rash with scarlet fever, but this rash is described as "sandpaperlike" with small papules, not vesicular, and is also not confined to the hands and feet.

Case 14: An 18-month-old girl presents to the clinic with nasal congestion and fever for three days. She is previously healthy. She is receiving acetaminophen for fever. Temperature is 101.2 F (38.4 C), pulse is 100 beats/minute, respirations are 24 breaths per minute. Oxygen saturations are 98% on room air. She is alert and well perfused with clear mucus coming from both nostrils and no increased work of breathing. Both turbinates show erythema. Her oropharynx is erythematous. No crackles or wheezing are heard. Her immunizations are up to date. Which of the following diagnoses is most consistent with this patient's presentation? A. Strep pnuemoniae B. Group A Strep C. Rhinovirus D. Hemophilus Influenzae type B E. Pertussis

The correct answer is C. A. Her symptoms do not fit a diagnosis of pneumonia, as she does not have significant fever, increased respiratory rate, crackles, or rales. B. Group A Strep is rarely a cause of pharyngitis in young children and notably does not cause the feared sequelae (rheumatic fever) in those < 3 years. C. Rhinovirus causes the common cold and is the most reasonable diagnosis. Rhinovirus is a very common cause of congestion and other cold-like symptoms. She presents with slightly elevated temperature, slight tachypnea, and inflamed turbinates and oral mucosa. Her symptoms all correlate with the common cold. D. Hemophilus Influenzae type B causes pneumonia and epiglottitis. She does not have the typical symptoms of epiglottitis that include difficulty breathing, high fever and drooling. With the advent of vaccinations, Hib infections have decreased significantly. E. The catarrhal phase of pertussis can be indistinguishable from the common cold, but quickly develops into the paroxysmal phase. The paroxysmal phase is characterized by coughing fits and post-tussive emesis, which again she does not have. It typically does not have associated fever.

Case 13: A 10-year-old boy comes to the clinic with a chief complaint of progressive cough for two weeks that began gradually. His cough is described as productive and wet with whitish sputum. His mother denies throat pain, vomiting, and diarrhea in his review of systems. His mother reports that he has been febrile up to 101.5°F daily. She thinks he is fatigued and has not eaten well in the past week. On exam, there is air passage throughout all lung fields, with crackles in the lower right lung field, but no other abnormal sounds. What would you likely find in your workup? A. Response to inhaled beta-agonist B. Hyperinflation in one lung field C. Alevolar consolidation in the RLL D. Positive PCR for pertussis E. Fluffy bilateral infiltrates and a large heart on chest x-ray

The correct answer is C. A. Response to an inhaled beta agonist is a good test for RAD or asthma. Asthma is diagnosed clinically, usually in a school-aged child, with a history of recurrent wheezing. Associated findings might atopic stigmata, such as allergic rhinitis, food allergy, and atopic dermatitis. Without details in the history, asthma is a less likely diagnosis in this setting, and in the absence of wheezing, asthma is even less likely. B. This would be consistent with a foreign body aspiration which could produce cough and fever (if bacterial superinfection occurs). This diagnosis is usually considered in younger children. Constitutional symptoms (fatigue, decreased eating) make pneumonia a better diagnosis in this clinical setting. C. Pneumonia is the most likely cause for his symptoms and a chest x-ray would be a great confirmation of your suspected diagnosis. Eliciting a complete history might reveal history of an upper respiratory infection. Localization of crackles (discontinuous inspiratory sounds) to one lobe makes pneumonia more likely. D. Pertussis can produce a lengthy cough illness, but is not associated with fever or lung findings. E. Pulmonary edema due to CHF is a symmetrical process and less likely to present with a unilateral lung finding. Pulmonary edema should be suspected with crackles, but this clinical setting leaves pulmonary edema low on our differential.

Case 11: A 3-year-old boy presents with fever to 103 F for the past week, injected eyes, and a refusal to walk for the past two days. On physical exam, you note conjunctival injection without pus or exudates bilaterally, prominent papillae of his tongue with redness as well as redness of his hands, and feet. He also has a new non-diffuse maculopapular rash on his torso that gets worse with fever. On examination of the swollen extremities, you are unable to elicit any tenderness or effusions in any joints. Which of the following is the most likely diagnosis? A. Rocky Mountain Spotted Fever (RMSF) B. Bone or joint infection C. Kawasaki disease (KD) D. Scarlet fever E. Systemic onset juvenile idiopathic arthritis

The correct answer is C. A. Rocky Mountain Spotted Fever (RMSF) is a tick-borne disease caused by Rickettsia rickettsii. This tick is commonly found in southeastern parts of the U.S., and patients will often come from or have a history of travel to that region. The disease is characterized by headache, fever, myalgia, and a centrally progressing petechial rash originating on the wrists and ankles. The maculopapular rash, and constellation of other symptoms, as well as lack of any recent travel history, makes this diagnosis less likely in this patient. B. Bone or joint infection should be on the differential given the patient's refusal to walk, as up to 80% of these infections are in the lower extremities. The fever associated with septic arthritis and osteomyelitis typically are not as elevated as the one presented in this case, and the lack of localized symptoms of warmth and tenderness associated with the lower extremity erythema and swelling make this diagnosis somewhat less likely in this patient. C. Kawasaki disease (KD) is one of the most common vasculitides of childhood. For diagnosis, in addition to fever of > 5 days, patient must meet four of the following criteria: rash, conjunctivitis, unilateral cervical lymphadenopathy, changes in oral mucosa, or extremity changes (redness/swelling). Our patient does not have lymphadenopathy, but often this is the least common finding in KD. If children have fever with fewer than four of the five clinical findings, they can have "incomplete KD" if they meet certain laboratory criteria. D. Scarlet fever is caused by erythrogenic toxin produced by Streptococcus pyogenes. Symptoms can include sore throat, fever, "strawberry tongue" and a blanching, erythematous rash with desquamation of the affected areas about six to seven days later as the rash begins to disappear. While our patient does have a "strawberry tongue," fever, and rash, the description of the rash and other physical exam findings are more consistent with KD than with scarlet fever. E. Systemic onset juvenile idiopathic arthritis, also known as Still's disease, is a subset of JIA describing patients with intermittent rash, fever and arthritis. While our patient does present with rash and fever, as well as refusal to walk (potentially a sign of arthritis), systemic onset JIA tends to present with a history of spiking fevers and "salmon" rash occurring when the child is febrile, and disappearing as the fever fades. This is inconsistent with the description of the findings seen in our patient, who does not demonstrate tenderness or effusion in any joint.

Case 13: A 9-year-old boy presents to your clinic with discoloration under his eyes, persistent cough, and skin rashes. He is found to have wheezing on physical exam and increased lung volume bilaterally on chest x-ray. He has struggled with these complaints over the past three years but recently his symptoms have gotten worse, affecting him every other day. He is afebrile. He is found to have wheezing on physical exam and increased lung volume bilaterally on chest x-ray. What would be the most appropriate treatment for him? A. Oral antibiotics B. Short-acting beta agonist PRN C. Short-acting beta agonist PRN with low-dose inhaled corticosteroid D. Short-acting beta agonist PRN with medium-dose inhaled corticosteroid E. Long-acting beta agonist

The correct answer is C. A. The patient's presentation is more consistent with asthma than an infection. The patient has had these complaints for the last few years. His skin rashes and lower eyelid darkening are consistent with allergic processes (atopy), which are associated with asthma. Furthermore, increased lung volumes bilaterally and persistent cough without fever also suggest asthma, thus antibiotics would not be appropriate. B. Cough and wheezing that occur intermittently (< 2 days/week) are consistent with intermittent asthma, which is treated with short-acting beta agonist PRN. C. Persistent cough and wheezing that affect the patient every other day (3-4 days with symptoms/week) are consistent with mild persistent asthma, which is appropriately treated with short-acting beta agonist PRN and low dose inhaled corticosteroid. The swelling under the eyes (allergic "shiners") and skin rash are other signs of atopy, as mentioned above. D. Short-acting beta agonist PRN with medium dose inhaled corticosteroid is the preferred treatment for moderate or severe persistent asthma, which corresponds to daily symptoms or symptoms throughout the day. E. The use of a long-acting beta agonist is reserved for severe persistent asthma, which corresponds to symptoms throughout the day.

Case 11: A 2-year-old girl presents to the urgent care clinic with a 7-day history of high fever to 38.5 C, a maculopapular rash that began on the palms and soles of her feet, red eyes without discharge, and unilateral cervical adenopathy. What other symptom/sign might you discover on further history and exam? A. Tonsillar exudates B. Headache C. Erythematous and edematous feet D. White spots on buccal mucosa E. Dysuria

The correct answer is C. A. Tonsillar exudates would be present in strep pharyngitis or tonsillitis. Given the prolonged fever, rash, lymph node involvement, and conjunctivitis, the disease process is more widespread than a simple tonsillitis. You should recognize this constellation of symptoms as Kawasaki disease. B. Headache would be present in Rocky Mountain Spotted Fever. This seems reasonable, given the fever and rash that began on the palms and soles. However, the other findings suggest Kawasaki disease, so this is not the best answer. C. The constellation of symptoms described suggests Kawasaki disease. The other two classic signs not mentioned are erythematous tongue ("strawberry tongue"), and erythema/edema of the extremities, which is the best answer here. D. White spots on the buccal mucosa are also known as Koplik spots, which are pathognomonic for measles. The fever and conjunctivitis could be measles, but there is no cough or coryza. In addition, the rash typically starts at the head and moves downward, rather than starting on the hands and feet. E. Although children with Kawasaki disease can have pyuria, it is not associated with dysuria, a symptom of a UTI, which would be highly unlikely given the other signs.

Case 10: A 10-day-old boy is brought to the ED by his mother because of "fever." Mom describes that the baby has been "sleepy" and feeding less vigorously than in the previous two days. She believes his urine output has also decreased. His birth history is notable for prolonged membrane rupture (about 32 hours), and maternal fever at the time of delivery. Prenatal and neonatal ultrasound revealed bilateral hydronephrosis. On exam, the infant is sleepy with a temperature of 38.5 C. A blood sample is sent for CBC, BMP, and culture. Attempts are made to obtain CSF and urine for analysis and culture, but only very small volumes of these fluids are obtained. Volume resuscitation is begun. Chest x-ray is performed with indeterminate results. What is the most appropriate next step? A. Send samples for gram stains and begin parenteral empiric antibiotic treatment B. Send the urine for urinalysis and the CSF for cell count, glucose and protein and begin parenteral antibiotic therapy C. Admit for observation and continue supportive care D. Send samples for culture and begin parenteral antiobiotic treatment E. Attempt to obtain larger samples. Antibiotics should not be started until all needed results are pending.

The correct answer is D. A. Although sending samples for gram stain may give an indication of whether an infection is present, it will not give the same degree of information as would a culture with sensitivities. B. Urinalysis and CSF profiles may help us make the diagnosis, but if positive in the absence of cultures, will commit us to a prolonged course of broad-spectrum and non-specific therapy. C. Delay of therapy would not be indicated. In an infant younger than one month, fever with any suspicion of sepsis, whatever the source, requires immediate evaluation and initiation of antibiotic treatment. D. Given the presentation of fever in a neonate who presents with sleepiness and poor feeding, samples should be sent for culture and the baby started on empiric antimicrobial therapy. This infant is likely to have a urinary tract infection, and urosepsis is certainly a possibility, especially given his known urinary tract anoamlies. We have no way of ruling out meningitis from this presentation, so antibiotics should be initiated at meningitic dosing. In an infant younger than one month, fever with any suspicion of sepsis, whatever the source, requires immediate evaluation and initiation of antibiotic treatment. Because infants at this age have immature immune systems, they do not localize infections as well as older children. An infection of the urinary tract may lead to bacteremia, which in turn may lead to CNS infection. Only cultures will give us the information required to determine the appropriate type length of antimicrobial therapy. E. Given the consequences of significant bacterial infection in an infant this age, delaying therapy to obtain additional laboratory specimens is not appropriate.

Case 11: A 5-year-old boy comes to the clinic with a chief complaint of four days of progressively worsening fever and that has been minimally responsive to acetaminophen. The patient complains of sore throat and decreased appetite. His sister had a positive rapid strep test and is now being treated with amoxicillin. Your concern is for Group A strep. What is the next best step in management? A. Start antibiotic treatment B. Send blood cultures C. Advise parents to give patient acetaminophen with return precautions D. Rapid strep test with back-up culture if negative E. Chest x-ray

The correct answer is D. A. Choice A is incorrect. Although you may empirically treat this child for infection with Group A strep, a test to diagnose infection should be done prior to initiation of antibiotics. B. Choice B is incorrect. There is no indication for a blood culture at this time. C. Choice C is incorrect. As the patient has a history of being exposed to a sick contact with Group A strep, being sent home with acetaminophen is also not sufficient because the patient has already been treated at home with acetaminophen with no improvement. D. Choice D would provide confirmation of your clinical suspicion and allow for correct diagnosis prior to empiric antibiotic treatment. E. Choice E is incorrect because there is no indication of respiratory symptoms.

Case 10: A 6-month-old female is brought into the pediatrician's office for three days of high fever, fussiness, and decreased appetite. The patient has not had any upper respiratory tract symptoms, vomiting, diarrhea, or rash. On physical exam the patient is fussy, has a RR of 28 bpm and a pulse of 160 bpm. She is febrile to 102.8 F (rectal). The patient is alert and fully moving all extremities. Apart from her vital signs, no other significant exam findings are noted. A CBC demonstrates leukocytosis of 17.0 cells x 103 / µL with elevated bands. What diagnosis is most likely? A. Measles B. Bacterial meningitis C. Acute otitis media D. Urinary tract infection E. Roseola

The correct answer is D. A. Measles typically begins with a "prodrome" period featuring the "3 Cs" (cough, coryza, conjunctivitis)-none of which this patient has-along with high fever, often > 104 F, and general malaise and anorexia. On the 2nd to 4th day a maculopapular erythematous rash appears starting on the face/upper neck and spreading downward. Although infants receive their first vaccination against measles (the MMR) at 1 year of age, infants are generally protected unless they are exposed to older, unimmunized children who have the disease. B. This answer is incorrect in this situation for several reasons. First, this patient is not toxic appearing, nor is her physical exam positive for any findings suggestive of meningitis (such as bulging fontanel or extreme irritability). Unlike older children or adults, classic meningeal signs will often not be present or will be difficult to appreciate on an infant. The patient's CBC is significant for leukocytosis with elevated bands, suggesting a bacterial infection. On exam the patient is alert and responding well to her environment and does not demonstrate lethargy, respiratory distress or signs of ICP. A definitive lumbar puncture may be ordered if there is more suspicion for meningitis or if the patient's status deteriorates in any way, and caretakers should be given a follow-up appointment as well as clear indications of when to seek care. C. This answer is incorrect because while fever and fussiness can be possible signs of AOM in infants, there is no evidence of infection on physical exam such as inflamed, erythematous tympanic membranes, with bulging of the membrane indicating an effusion. D. UTI, the most common bacterial illness in a female infant, is consistent with her high fever, fussiness, and decreased appetite. Her CBC suggests that she has a bacterial infection (leukocytosis and elevated bands). A sample of her urine should be obtained by catheterization and sent for urinalysis and culture. E. Roseola often presents with a high fever, but also often with a viral prodrome. It is a diagnosis of exclusion at this point and should not preclude obtaining a urine sample in this child.

Case 16: A 15-year-old female is brought to the ER abdominal pain for 3 hours. She rates her pain at 8/10 and describes it as constant and located mainly in the middle of her belly, but somewhat present throughout her abdomen. It is worse with coughing and moving. She had two episodes of nonbilious, non-bloody vomiting. She is sexually active with her boyfriend of three months and always uses condoms. Her last menstrual period was two weeks ago. Vital signs are normal. On exam, she exhibits involuntary guarding, mild rebound tenderness, and tenderness to palpation between her right anterior superior iliac spin and umbilicus. On pelvic exam, she reports tenderness when attempting to palpate her right adnexa, but no masses are appreciated and there is no cervical motion tenderness. Laboratory studies reveal normal complete blood count and creactive protein. Which of the following is the most likely diagnosis in this patient? A. Ovarian torsion B. Pelvic inflammatory disease C. Ectopic pregnancy D. Appendicitis

The correct answer is D. A. Ovarian torsion is more common in the post-menopausal population, though it can present in any age group. It is described as intermittent stabbing pain in the lower abdomen or pelvis. Torsion is often secondary to an ovarian mass, such as a neoplasm or corpus luteal cyst, which may occasionally be appreciated on exam. Nausea and vomiting are very common findings as well. Ultrasound is essential to initial workup. Given the patient's pain localized around her umbilicus, her tenderness at McBurney's point, and lack of palpable masses on pelvic exam, ovarian torsion is a less likely diagnosis. B. Pelvic inflammatory disease is definitely a possibility given the patient's sexual history, and adnexal pain. However, this pain is often post-coital and also often occurs during or immediately following menstruation. Another key finding is mucopurulent discharge and cervical motion tenderness, both of which are absent in this patient. RUQ pain and a fever (present in 50% of patients with PID) are other signs that are not reported in this case. C. Patients with an ectopic pregnancy typically present with painless vaginal bleeding six to eight weeks after their last menstrual period. The pain is described as crampy pelvic pain, and it is often associated with nausea. Diffuse abdominal pain is also present if rupture and intraperitoneal bleeding occurs. Given that the patient is two weeks from her last menstrual period, she reports no vaginal bleeding, and her pain is intense and located in the middle of her abdomen, an ectopic pregnancy is less likely. D. Appendicitis is the most common condition in pediatric patients requiring immediate surgical intervention, but often (especially in infants) presents differently than in adults. Aspects of their atypical presentation include lack of migration of pain to the RLQ, negative Rovsing's sign, and involuntary guarding and fever without perforation. School-age children who can articulate the pain often describe pain with movement or coughing. Also, rebound tenderness was found to be neither sensitive nor specific in the pediatric population, while in the adult population it is one of the most accurate PE findings (86%). This patient's sudden onset of intense pain at the umbilicus with vomiting, anorexia, and tenderness at McBurney's point are all classic findings of appendicitis. The more atypical signs include diffuse pain centered below the umbilicus, and rebound tenderness that might point to a perforation (more likely, it is part of the atypical pediatric presentation given her normal WBC). Another atypical aspect of her exam is her adnexal pain during the pelvic exam, which could be due to the degree of inflammation and the positioning of her appendix. The key take-away point is to have a high index of suspicion for appendicitis in pediatric patients with abdominal pain given their atypical presentation.

Case 14: A 14-month-old female presents to clinic with fever to 39.2 C and irritability. According to mom, the patient was initially sick one week ago with a runny nose and cough, but these symptoms had resolved. She started pulling at her ear and becoming increasingly irritable last night. She has had several prior ear infections, and was most recently treated one month ago with amoxicillin. She is up to date on immunizations. Physical examination reveals a red, opaque, bulging tympanic membrane with bubbles and limited mobility of her left ear. The exam of the right ear is normal. Which of the following is the next step in management of this patient? The best option is indicated below. Your selections are indicated by the shaded boxes. A. Observation B. Anthistamines and decongestants C. High-dose amoxicillin D. Amoxicillin/clavulanate E. Tympanocentesis

The correct answer is D. A. This choice is incorrect due to the patient's presentation with a high fever above 39 C. Fifty to 80% of acute otitis media cases will resolve spontaneously without antibiotics; however, the decision to defer treatment with the "observation option" is based on the child's age and illness severity. This option is limited to healthy children between the ages of 6 months to 2 years with non-severe symptoms. Our patient is presenting with severe symptoms and a high-grade fever. B. This choice is incorrect because the FDA has discouraged the use of over-the-counter cough and cold products in children younger than 2 years due to the increased risk/benefit ratio. Also, these medications would be most useful for the upper respiratory symptoms preceding the ear infection. C. High-dose amoxicillin is the most common first-line treatment for acute otitis media due to its general effectiveness against susceptible and partially resistant S. pneumo, in addition to being low cost and having a high safety profile. However, this antibiotic was recently administered, raising concerns for a resistant organism. D. This choice is correct because of the severe symptoms our patient is exhibiting with a high temperature greater than 39 C. Amoxicillin/clavulanate is the treatment of choice for patients with moderate to severe otalgia or high fever, and is used for additional beta-lactamase coverage for Haemophilus influenzae and Moraxella catarrhalis, and when failure with amoxicillin is suspected. E. This choice is incorrect because the patient has not had recurrent episodes of otitis media and has not started antibiotic therapy. Tympanocentesis is recommended as a diagnostic measure to confirm a bacterial etiology after a patient has failed repeated courses of antibiotics or if an unusually resistant organism is suspected.

Case 13: A 4-year-old patient presents with several months of cough. Mom also reports a history of red skin patches, which are pruritic, and allergies to peanuts, eggs, and mangoes. Which of the following would be characteristic of the cough that this patient would present with? A. Does not awaken patient from sleep B. Paroxysmal C. Barking cough D. Worse at night E. Associated with crackles on exam

The correct answer is D. A. This choice is incorrect. This patient has asthma, which commonly presents with symptoms awakening the patient from sleep. Habitual cough disappears at night. B. This choice is incorrect. Paroxysmal coughs are associated with bacterial infections such as pertussis, Chlamydia, or mycoplasma. Foreign bodies can also produce sudden onset of cough. History could help to determine if the latter is a cause. C. This choice is incorrect. Barking coughs are associated with croup or other forms of subglottic disease. Foreign bodies can also produce this type of cough. D. This choice is correct. Asthma frequently presents with nighttime exacerbations. The cough often presents with wheezing and is usually a dry cough. E. This choice is incorrect. Diseases associated with crackles usually have intrinsic pulmonary involvement. Crackles can be fine or coarse and usually represent alveolar or small airway conditions.

Case 7: A 2-hour-old infant is evaluated in the nursery for progressively worsening tachypnea. He was born at 32 weeks' gestational age via spontaneous vaginal delivery to a healthy mother with negative group B streptococcus status. There was no premature rupture of membranes and no meconium in the amniotic fluid. His Apgar scores were 8 at one minute and 9 at five minutes. On physical examination he is large for gestational age. His vital signs are respiratory rate 75, temperature 36.5 C and heart rate is 130 beats per minute. His lung exam is remarkable for intercostal and subcostal retractions, grunting, and equal breath sounds. His heart exam reveals normal rhythm, normal S1 and S2, no murmurs, and normal peripheral pulses and capillary refill. Which of the following is the most likely cause of the patient's condition? A. Transient tachypnea of the newborn (TTN) B. Pneumothorax C. Congestive heart failure D. Respiratory distress syndrome E. Sepsis

The correct answer is D. A. Transient tachypnea of the newborn (TTN) is much more common in infants born to diabetic mothers. TTN is unlikely because he is 32 weeks, very premature, and was born via NSVD. RDS is much more likely, although TTN is still a possibility and would need to evaluated with a CXR. B. Pneumothoraces are uncommon, but should always be considered in an infant with respiratory distress. Good bilateral air entry argues against this diagnosis. C. Congestive heart failure is an important cause of tachypnea. Adam has a normal cardiovascular exam, with no murmur, normal pulses, and good capillary refill. Infants with congestive heart failure usually present with the triad of tachypnea, tachycardia, and hepatomengaly. D. Respiratory distress syndrome (RDS) causes tachypnea and is therefore an important consideration in this case. RDS is more common in premature infants. Given the lack of history of maternal diabetes, an NSVD birth, and few risk factors for sepsis other than prematurity, Adam is likely to have RDS. E. Infants may present with respiratory distress from sepsis or from pneumonia. In this case, Baby Adam has a normal temperature, good blood pressure, and normal perfusion. While less likely, this diagnosis should always be considered in infants with respiratory distress.

Case 15: Johnny is a 25-month-old male who presents to the ED with a 2-day history of vomiting and diarrhea. Dad relays a history of abrupt onset of vomiting that started yesterday around 1 pm. Johnny has had 6 episodes of emesis since yesterday and 3 episodes of diarrhea. The emesis is non-bilious and the diarrhea is described as watery with specks of blood throughout the diarrhea. There are no sick contacts in the home. Vital signs: T 37.1, P 102, R 20, BP 90/60. Physical examination is normal and Johnny has still been tolerating some PO feeds without instant vomiting. What is the most immediate intervention for this patient? A. IV bolus with D5W B. IV bolus with 0.9% saline C. CT scan and surgical consult D. random glucose test E. no immediate intervention is necessary

The correct answer is E. A. An IV bolus with D5W is indicated in cases of confirmed hypoglycemia and is used for maintenance fluids. This is not indicated in this patient. B. With his normal vitals and no obvious signs of dehydration, an IV bolus of 0.9% saline is not indicated in this patient. If there was evidence of dehydration on physical exam or with vitals, then IV fluids would be necessary. C. This vignette does not seem like a surgical case. Physical examination was normal, which makes abdominal pathology less likely. While appendicitis might be in the differential diagnosis, other diagnoses are more likely, making the excessive radiation exposure from a CT scan not necessary. D. With cases of dehydration one must always think about abnormalities in blood glucose levels. This clinical presentation does not suggest signs of hypo or hyperglycemia. E. At this point the patient is most likely suffering from a case of viral gastroenteritis. Because he is still tolerating some PO feeds, has no obvious signs of dehydration, and has normal vital signs, there is no need for aggressive IV fluid administration or diagnostic work up. Strict return precautions should be given and it should be advised that Johnny maintains fluids as much as possible.

Case 16: A 4-year-old girl with a history of type 1 diabetes mellitus was admitted to a local hospital for treatment of DKA. A few hours after the treatment, she develops grunting, irregular respirations, and has vomited twice. On exam, her left eye is pointing downward and out on straight gaze. Her diastolic blood pressure is 90 mmHg. What is a likely diagnosis? A. Hypoglycemia B. Hypokalemia C. Hyponatremia D. Pneumonia with possible sepsis E. Cerebral edema

The correct answer is E. A. This choice is incorrect because although hypoglycemia can be a complication of DKA treatment, it would be unlikely to cause a cranial nerve palsy. B. This choice is incorrect. Although hypokalemia can be a complication of DKA treatment, it would likely present as an elevated BP, muscular weakness or myalgia, as well as muscle cramps, constipation, and hyporeflexia in severe cases, rather than with the symptoms described in this vignette. C. This choice is incorrect because, while hyponatremia can occur (due to the dilutional effect caused by water shifting from the intracellular to the extracellular compartment because of hyperglycemia and increased plasma osmolarity), it would be corrected with the DKA treatment and would not present with a cranial nerve palsy. D. This choice is incorrect because pneumonia would not present with a cranial nerve palsy, although it might be important to evaluate patients with DKA for signs of intercurrent illness, including pneumonia, UTI, and perinephric abscess. E. This choice is correct. Administration of bicarbonate during DKA treatment increases the risk of cerebral edema. Although symptomatic cerebral edema is rare (less than 1%), it is associated with a high mortality rate (over 20%). The signs of cerebral edema are described in the vignette, and include irregular respirations, headache, vomiting, third nerve palsy, and high blood pressure.


Set pelajaran terkait

Shape, Size, Structure of the Earth

View Set

Human Resource Management Ch 14, 15, & 16

View Set

WGU Wiley Quizzes Intermediate Accounting I

View Set